Anatomy Unit 2 BRS/Lippincott's Upper Limb

¡Supera tus tareas y exámenes ahora con Quizwiz!

The answer is A. This is the flexor pollicis longus, which is innervated by the median nerve.

71. A lesion of the median nerve causes paralysis of this structure.

The answer is C. The ulnar nerve innervates the two medial lumbricals. However, the median nerve innervates the two lateral lumbricals, the flexor digitorum superficialis, the opponens pollicis, and the pronator teres muscles.

28. A 27-year-old man with cubital tunnel syndrome complains of numbness and tingling in the ring and little finger and back and sides of his hand because of damage to a nerve in the tunnel at the elbow. Which of the following muscles is most likely to be paralyzed? (A) Flexor digitorum superficialis (B) Opponens pollicis (C) Two medial lumbricals (D) Pronator teres (E) Supinator

The answer is D. Winged scapula is caused by paralysis of the serratus anterior muscle that results from damage to the long thoracic nerve, which arises from the roots of the brachial plexus (C5-C7).

3. A 29-year-old man comes in with a stab wound, cannot raise his arm above horizontal, and exhibits a condition known as "winged scapula." Which of the following structures of the brachial plexus would most likely be damaged? (A) Medial cord (B) Posterior cord (C) Lower trunk (D) Roots (E) Upper trunk

The answer is C. To hold a typing paper, the index finger is adducted by the palmar inter- osseous muscle, and the middle finger is abducted by the dorsal interosseous muscle. Both muscles are innervated by the ulnar nerve.

30. A man is unable to hold typing paper between his index and middle fingers. Which of the following nerves was likely injured? (A) Radial nerve (B) Median nerve (C) Ulnar nerve (D) Musculocutaneous nerve (E) Axillary nerve

The answer is D. The dorsal interossei are abductors of the fingers. The little finger has no attachment for the dorsal interosseous muscle because it has its own abductor. Therefore, the dorsal interosseous muscle is not affected. Other muscles are attached to the little fin- ger, thus they are injured.

38. A 24-year-old carpenter suffers a crush injury of his entire little finger. Which of the following muscles is most likely to be spared? (A) Flexor digitorum profundus (B) Extensor digitorum (C) Palmar interossei (D) Dorsal interossei (E) Lumbricals

The answer is B: Ulnar nerve in the wrist. The ulnar nerve can become compressed between the pisiform and hook of the hamate at the wrist in a condition termed "ulnar canal syndrome" or "Guyon tunnel syndrome." This entrapment syndrome is especially seen in professional cyclists who spend countless hours placing pressure on the hook of the hamate bone as they grasp their handlebars. This "handlebar neuropathy" presents with hyperextension of the metacarpophalangeal joints and flexion at the interphalangeal joints of the fourth and fifth fingers. The "clawing" of these two fingers is accompanied by sensory loss in the medial side of the hand. Choice A (Ulnar nerve in the elbow) is incorrect. The ulnar nerve crosses the elbow in a narrow space between the olecranon process and the medial epicondyle of the humerus, on the posteromedial (ulnar) side of the joint. Compression of the ulnar nerve at this location leads to "cubital tunnel syndrome" and presents with hand deficits similar to those seen in this patient. However, this patient does not exhibit weakness in flexion or adduction of the wrist. Sparing of these actions indicates that damage to the ulnar nerve must be distal to the elbow. Choice C (Median nerve in the wrist) is incorrect. The median nerve is often compressed at the wrist in "carpal tunnel syndrome" because it travels deep to the transverse carpal ligament (flexor retinaculum of the wrist) within the carpal tunnel. This entrapment syndrome presents with paresthesia of the lateral fingers as well as an inability to oppose the thumb and a wasting of the thenar eminence. Damage to the median nerve would not lead to the postural deformity seen in the medial two fingers in this patient. Choice D (Median nerve in the elbow) is incorrect. The median nerve is not usually compressed by forced flexion of the elbow because it lies loosely on the flexor surface of the joint deep to the bicipital aponeurosis. The median nerve can be compressed in the proximal forearm as it passes between the two heads of the pronator teres. However, damage to the median nerve would not lead to this patient's postural deformity in the medial two fingers. Choice E (Median nerve in the axilla) is incorrect. The median nerve would not be damaged in the axilla without significant trauma. Damage to the median nerve in the axilla would lead to weakness in flexing the wrist, loss of pronation of the forearm, and wasting of the thenar muscles. However, these signs and symptoms were not reported."

50. A 52-year-old retired professional cyclist, who still rides his bike 400 miles per week, comes to his physician complaining of hand problems. The physician notes hyperextension of the ring and little fingers at the metacarpophalangeal joints and flexion at the interphalangeal joints within the same fingers (see photo). During examination, the patient has no weakness in flexion or adduction of the wrist. What nerve is compressed at what location? (A) Ulnar nerve in the elbow (B) Ulnar nerve in the wrist (C) Median nerve in the wrist (D) Median nerve in the elbow (E) Median nerve in the axilla

The answer is G. This is the third palmar interosseous muscle, which adducts the little finger.

72. The patient is unable to adduct her little finger because of paralysis of this structure.

The answer is C: Anterior interosseous nerve. The anterior interosseous nerve is a branch of the median nerve in the distal part of the cubital fossa, and it courses distally on the interosseous membrane. It supplies the deep forearm flexors, including the flexor digitorum profundus of digits 2 and 3, the flexor pollicis longus, and the pronator quadratus. Loss of this nerve would cause weakness in pronation due to denervation of the pronator quadratus. This injury would also result in inability to flex the distal interphalangeal joints of the index and middle fingers and the interphalangeal joint of the thumb due to denervation of the flexor digitorum profundus and flexor pollicis longus, respectively. This deficit would lead to the collapsed "O.K. sign" indicated in the photo. Because compromising the anterior interosseous nerve would not result in any cutaneous sensory deficits, it is this nerve that was most likely damaged by the displaced end of the left radius. Choice A (Deep branch of the radial nerve) is incorrect. The deep branch of the radial nerve arises from the radial nerve in the cubital fossa, dives deep to pierce the supinator muscle, and supplies all the muscles in the posterior compartment of the forearm. Damage to this nerve would lead to "wrist drop" in the patient due to the flexors of the forearm being unopposed and an inability to extend at the wrist. Choice B (Superficial branch of the radial nerve) is incorrect. The superficial branch of the radial nerve arises from the radial nerve in the cubital fossa. This nerve is entirely a sensory nerve, supplying the dorsum of the hand and fingers. Because the patient has motor deficits, this nerve was not damaged by the displaced radius. Choice D (Median nerve proximal to the carpal tunnel) is incorrect. The median nerve proximal to the carpal tunnel would not be injured because damaging this nerve would lead to a significant sensory loss over the thumb and the adjacent two and a half fingers in addition to motor loss to the thenar eminence (opponens pollicis, abductor pollicis brevis, and flexor pollicis brevis muscles) and the first two lumbrical muscles. So, the median nerve was not injured proximal to the carpal tunnel. Choice E (Recurrent [thenar] branch of median nerve) is incorrect. The recurrent (thenar) branch of the median nerve innervates most of the thenar muscles, including the abductor pollicis brevis, opponens pollicis, and superficial head of the flexor pollicis brevis. Cutting this nerve would lead to atrophy of the thenar muscular complex, a condition known as "ape hand." In this condition, the patient would not be able to oppose the thumb and second finger. Though this nerve has no sensory distribution, it is unlikely that it was involved due to the weakness when pronating the forearm.

11. A 23-year-old man was injured in a motor vehicle accident and X-rays confirmed a displaced distal radius fracture in his left forearm. Upon examination, the patient exhibits weakened pronation, weakened flexion of the index and middle fingers at the distal interphalangeal joints, and weakened flexion of the interphalangeal joint of the thumb. When asked to make the "okay" sign (make a circle with the thumb and index finger), the patient is unable to make a round circle, producing a "col- lapsed circle" on the affected hand (see photo). No areas of sen- sory loss are detected. Which nerve is most likely damaged? (A) Deep branch of the radial nerve (B) Superficial branch of the radial nerve (C) Anterior interosseous nerve (D) Median nerve, proximal to the carpal tunnel (E) Recurrent branch of the median nerve

The answer is C. The extensor digitorum, flexor digitorum profundus, dorsal interosseous, and lumbrical muscles are attached to the middle digit, but no palmar interosseous mus- cle is attached to the middle digit.

12. An automobile body shop worker has his middle finger crushed while working on a transmission. Which of the following muscles is most likely to retain function? (A) Extensor digitorum (B) Flexor digitorum profundus (C) Palmar interosseous (D) Dorsal interosseous (E) Lumbrical

The answer is B: Lunate. The lunate is shaped like a moon, thus its name. It is situated in the center of the proximal row of carpal bones where it articulates with the radius. This bone is the most commonly dislocated carpal bone, which leads to severe carpal instabilities. This dislocation often occurs in association with a trans-scaphoid fracture. It is important to note that scaphoid fractures are often difficult to see in radio- graphic imaging; however, these fractures can be detected fre- quently by applying direct pressure to the anatomical snuff- box. Choice A (Scaphoid) is incorrect. The scaphoid bone is located in the proximal row and is the most frequently frac- tured carpal bone. As the weight of the body is transmitted through the upper limb onto the outstretched hand and the impact of the fall exerts pressure back upon the limb, the scaphoid bone is crushed by these opposing forces and is subsequently fractured, usually along its narrowest part. The scaphoid bone is clinically relevant due to the frequency of fractures in younger patients and the poor vascularization of its proximal part. Avascular necrosis is often a postfracture complication that slows the healing of this bone. Choice C (Capitate) is incorrect. The capitate is located in the distal row of carpal bones, so this selection can be easily eliminated. Choice D (Triquetrum) is incorrect. The triquetrum is located in the proximal row of carpal bones and may be involved in a severe dislocation of the wrist. Specifically, the triquetrum can be displaced in Stage III wrist dislocations when the triquetrol- unate interosseous ligament is damaged. However, tearing this interosseous intercarpal ligament would only occur following dislocation of the lunate bone, which is the most commonly dislocated carpal bone. Choice E (Trapezium) is incorrect. The trapezium bone helps form the distal floor of the anatomic snuffbox and forces would be exerted on this bone during the fall. However, it is not located in the proximal row of carpal bones. Moreover, this four-sided bone is rarely fractured or dislocated due to its shape and construction.

14. A 19-year-old man arrives at his campus health clinic complaining of soreness in his right wrist. He explains he landed on an outstretched hand when he was tackled in a rugby match. He indicates that the pain worsens with movement and is minimized by stabilization of the wrist. There are no sensory deficits in his hand nor does he have trouble grasping or holding objects. Pressure applied to the anatomic snuffbox between the extensor pollicis brevis and extensor pollicis longus tendons produces no pain. Radiographic studies show no fractures but reveal an anterior dislocation of a bone in the proximal row of carpal bones. What carpal bone is most likely dislocated in this patient? (A) Scaphoid (B) Lunate (C) Capitate (D) Triquetrum (E) Trapezium

The answer is D. Structures entering the palm superficial to the flexor retinaculum include the ulnar nerve, ulnar artery, palmaris longus tendon, and palmar cutaneous branch of the median nerve. The median nerve, the flexor pollicis longus, and the flexor digitorum superficialis and profundus run deep to the flexor retinaculum.

15. A man injures his wrist on broken glass. Which of the following structures entering the palm superficial to the flexor retinaculum may be damaged? (A) Ulnar nerve and median nerve (B) Median nerve and flexor digitorum profundus (C) Median nerve and flexor pollicis longus (D) Ulnar artery and ulnar nerve (E) Ulnar nerve and flexor digitorum superficialis

The answer is E. The superior thoracic artery is a direct branch of the axillary artery. The thoracoacromial trunk has four branches: the pectoral, clavicular, acromial, and deltoid.

2. A patient comes in with a gunshot wound and requires surgery in which his thoracoacromial trunk needs to be ligated. Which of the following arterial branches would maintain normal blood flow? (A) Acromial (B) Pectoral (C) Clavicular (D) Deltoid (E) Superior thoracic

The answer is D: Biceps brachii. The lateral X-ray reveals chronic microtrauma to the radial tuberosity, which is the insertion site for the biceps brachii muscle. In this weightlifter, the damage to the radial tuberosity is most likely due to overuse of the biceps brachii and the concomitant stress placed upon the radial tuberosity by lifting significant weight loads. The biceps brachii muscle produces flexion of the elbow (and shoulder) and is the powerful supinator of the forearm, explaining why these actions exacerbated the pain in this patient. Choice A (Supinator) is incorrect. The supinator muscle attaches onto the proximal shaft of the radius rather than the radial tuberosity, so it would not cause damage to the radial tuberosity. Moreover, the supinator muscle acts to produce lower resistance supination and does not flex the elbow. Choice B (Brachialis) is incorrect. The brachialis muscle is a powerful flexor of the elbow. However, it is not related to the radial tuberosity as it attaches onto the coronoid process and tuberosity of the ulna. Furthermore, the brachialis muscle does not act in supination. Choice C (Pronator teres) is incorrect. The pronator teres muscle contributes somewhat to flexion of the elbow. However, it inserts onto the midshaft portion of the radius, and its main action is pronation. In this patient, pain was exacerbated by supination against resistance. Choice E (Brachioradialis) is incorrect. The brachioradials muscle is a notable flexor of the elbow. However, it attaches to the styloid process at the distal end of the radius and is not related to the radial tuberosity. Also, the brachioradialis muscle does not act in supination.

22. A 23-year-old competitive weight lifter goes to his physician complaining of pain in his proximal forearm. During his examination, the pain is exacerbated by flexion of the elbow and supination of the forearm against resistance. A lateral radiograph shows chronic microtrauma to the proximal radius, marked by the black arrows. Which of the following muscles attaches to, and most likely damaged, this osteological process? (A) Supinator (B) Brachialis (C) Pronator teres (D) Biceps brachii (E) Brachioradialis

The answer is B. The brachialis and supinator muscles form the floor of the cubital fossa. The brachioradialis and pronator teres muscles form the lateral and medial boundaries, respectively. The pronator quadratus is attached to the distal ends of the radius and the ulna.

22. A 27-year-old baseball player is hit on his forearm by a high-speed ball during the World Series, and the muscles that form the floor of the cubital fossa appear to be torn. Which of the following groups of muscles have lost their functions? (A) Brachioradialis and supinator (B) Brachialis and supinator (C) Pronator teres and supinator (D) Supinator and pronator quadratus (E) Brachialis and pronator teres

The answer is B. The medial brachial cutaneous nerve contains sensory (general somatic afferent [GSA]) fibers that have cell bodies in the dorsal root ganglia, and an injury of these GSA fibers causes numbness of the medial side of the arm. It also contains sympathetic postganglionic fibers that have cell bodies in the sympathetic chain ganglia. The anterior horn of the spinal cord contains cell bodies of skeletal motor (general somatic efferent [GSE]) fibers, and the lateral horn contains cell bodies of sympathetic preganglionic fibers. The posterior horn contains cell bodies of interneurons.

23. A 23-year-old man complains of numbness on the medial side of the arm following a stab wound in the axilla. On examination, he is diagnosed with an injury of his medial brachial cutaneous nerve. In which of the following structures are the cell bodies of the damaged nerve involved in numbness located? (A) Sympathetic chain ganglion (B) Dorsal root ganglion (C) Anterior horn of the spinal cord (D) Lateral horn of the spinal cord (E) Posterior horn of the spinal cord

The answer is E: Teres minor. Throwing motions are complex mechanical events that involve multiple muscles interacting in moment-to-moment changing ways, with rotation of the humerus being one important outcome. The four rotator cuff muscles (supraspinatus, infrapsinatus, teres minor, subscapularis), plus several other muscles that cross the glenohumeral joint, contribute significant forces to rotation of the arm. The teres minor and infraspinatus (i.e., half the rotator cuff group) lie completely across the posterior aspect of the glenohumeral joint and are primary lateral (external) rotators. These muscles are aided by the posterior fibers of the deltoid muscle. Choice A (Supraspinatus) is incorrect. The supraspinatus is one of the rotator cuff muscles. However, it does not produce rotation. It lies across the superior aspect of the glenohumeral joint and initiates abduction of the arm from the rest position. Choice B (Teres major) is incorrect. The teres major arises posteriorly from the inferior angle of the scapula and crosses the glenohumeral joint to its anteriorly located insertion into the medial lip of the intertubercular groove of the humerus. Thus, the teres major muscle acts to adduct and medially (internally) rotate the arm rather than laterally rotate. It is not a member of the rotator cuff muscles. Choice C (Latissimus dorsi) is incorrect. The latissimus dorsi muscle arises posteriorly from the spinous processes of the inferior six thoracic vertebrae and the thoracolumbar fascia and crosses the glenohumeral joint anteriorly to insert into the floor of the intertubercular sulcus of the humerus. Thus, the latissimus dorsi muscle acts to extend, adduct, and medially rotate the humerus rather than laterally rotate. It is not a member of the rotator cuff muscles. Choice D (Subscapularis) is incorrect. The subscapularis muscle is part of the rotator cuff. In common with the teres major and latissimus dorsi muscles, it crosses the glenohumeral joint from posterior to anterior, thus producing medial rotation and adduction of the arm. It is not responsible for external (lateral) rotation of the arm.

26. Lateral rotation of the arm is an important mechanical component of "bringing the arm back" when preparing to throw an object. What muscle acts to produce lateral rotation of the arm? (A) Supraspinatus (B) Teres major (C) Latissimus dorsi (D) Subscapularis (E) Teres minor

The answer is B: Axillary artery. The brachial plexus is divided into five geographic parts: Roots (or ventral rami of C5-T1), Trunks, Divisions, Cords, terminal Branches (or Nerves). These sections of the brachial plexus can be remembered with the mnemonics, "Real Truckers Drink Cold Beer" or "Remember Those Darn Cervical Nerves". The cords are named according to their important positional relationship to the second part of the axillary artery, deep to the pectoralis minor muscle. Here, the nerves form a cradle-like bed for this segment of the vessel as it passes through the axilla. Thus, the lateral cord is located lateral, the medial cord is medial, and the posterior cord is posterior to the axillary artery. Choice A (Long head of the biceps brachii muscle) is incorrect. The long head of the biceps brachii muscle lies lateral to all three cords of the brachial plexus. Choice C (Subclavian vein) is incorrect. This vessel is located well proximal to the cords of the brachial plexus, medial to the first rib. Choice D (Surgical neck of the humerus) is incorrect. This part of the humerus is located distal and lateral to all three cords of the brachial plexus. Choice E (Pectoralis minor muscle) is incorrect. The pectoralis minor lies superficial (anterior) to the cords of the brachial plexus. This muscle divides the axillary artery into its three parts. Thus, it defines the second part of the artery and creates the situation for naming the cords of the plexus.

27. The lateral cord of the brachial plexus is named because it lies immediately lateral to which of the following structures? (A) Long head of the biceps brachii muscle (B) Axillary artery (C) Subclavian vein (D) Surgical neck of the humerus (E) Pectoralis minor muscle

The answer is C: Lateral to the tendon of the flexor carpi radialis. The most common location for measuring pulse rate is on the radial artery at the wrist. Here, the vessel lies on the anterior side of the distal end of the radius, lateral to the tendon of the flexor carpi radialis. It is covered only by skin and a thin superficial fascia and can be palpated easily against the radius. Note the placement of the index and middle fingers of the physician in the given photo. Choice A (Across the anterior aspect of the lateral epicondyle of the humerus) is incorrect. The radial artery originates in the cubital fossa as one of the two terminal branches of the brachial artery (the ulnar artery is the other). The radial recurrent artery branches off the radial artery just below its origin and ascends across the anterior aspect of the lateral epicondyle of the humerus. However, the radial recurrent artery lies on muscle and is not normally palpable. Choice B (Between the tendons of the palmaris longus and flexor carpi ulnaris) is incorrect. The ulnar artery enters the hand superficial to the flexor retinaculum, lateral to the pisiform and medial to the hook of the hamate. The ulnar pulse may be palpable slightly lateral to the insertion of the flexor carpi ulnaris onto the pisiform, between it and the tendon of the palmaris longus. Choice D (Superficial to the tendons of the extensor pollicis brevis and abductor pollicis longus) is incorrect. The radial pulse is also available in the anatomical snuffbox, where the radial artery crosses the floor of that space between the tendons of the extensor pollicis longus and brevis muscles. In entering the snuffbox, the artery passes deep (not superficial) to the tendons of the abductor pollicis longus and extensor pollicis brevis muscles. Choice E (Superficial to the carpal tunnel) is incorrect. As noted above, the ulnar artery enters the hand superficial to the flexor retinaculum, thus, superficial to the carpal tunnel. The superficial palmar branch of the radial artery usually runs through the thenar muscles and is not palpable.

29. The pulse of the radial artery is readily palpable where the vessel passes which of the following structures? (A) Across the anterior aspect of the lateral epicondyle of the humerus (B) Between the tendons of the palmaris longus and flexor carpi ulnaris (C) Lateral to the tendon of the flexor carpi radialis (D) Superficial to the tendons of the extensor pollicis brevis and abductor pollicis longus (E) Superficial to the carpal tunnel

The answer is D: Radial artery. The radial and ulnar arteries anastomose to form the superficial and deep palmar arches. However, the superficial palmar arch is formed mainly by the ulnar artery, whereas the deep palmar arch is formed primarily by the radial artery. Thus, the insufficiency in this case is most likely due to reduced flow in the radial artery. Choice A (Posterior interosseous artery) is incorrect. The posterior interosseous artery branches off the common interosseous artery, which is derived from the ulnar artery in the proximal forearm. The posterior interosseous artery supplies the posterior compartment of the forearm. This artery does not reach the wrist and does not contribute blood to the palmar arterial arches. Choice B (Ulnar artery) is incorrect. The ulnar artery provides the primary supply into the superficial palmar arterial arch. The deep palmar arch is formed primarily by the radial artery, and it is the reduced flow of the radial artery most likely present in this patient. Choice C (Anterior interosseous artery) is incorrect. The anterior interosseous artery branches off the common interosseous artery, which is derived from the ulnar artery in the proximal forearm. The anterior interosseous artery supplies the anterior compartment of the forearm as well as the distal posterior forearm via its posterior terminal branch. This artery does reach the wrist, but it does not contribute blood to the palmar arterial arches. Choice E (Inferior ulnar collateral artery) is incorrect. This artery branches off the brachial artery just above the elbow. It passes distally across the anterior side of the medial epicondyle of the humerus to form a collateral connection with the anterior ulnar recurrent branch of the ulnar artery. This vessel is far removed from the hand.

30. A 74-year-old man complains of pain in his right hand and fingers when he works with his hands for a while. Thorough testing reveals insufficient blood flow into the deep palmar arch. Occlusion of which of the following arteries is the most likely cause of this condition? (A) Posterior interosseous artery (B) Ulnar artery (C) Anterior interosseous artery (D) Radial artery (E) Inferior ulnar collateral artery

The answer is D. The proximal row of carpal bones consists of the scaphoid, lunate, tri- quetrum, and pisiform bones, whereas the distal row consists of the trapezium, trapezoid, capitate, and hamate bones.

31. The victim of an automobile accident has a destructive injury of the proximal row of carpal bones. Which of the following bones is most likely damaged? (A) Capitate (B) Hamate (C) Trapezium (D) Triquetrum (E) Trapezoid

The answer is E: Cephalic vein. The great saphenous vein is a large cutaneous vessel that ascends the medial aspect of the lower limb to ultimately drain into the femoral vein at the top of the limb. The cephalic vein is a distinctive cutaneous vessel that ascends the lateral aspect of the arm to ultimately drain into the axillary vein. At first glance, these two veins might seem to be traveling opposite venous routes. However, notice that each vein is aligned along the first digit side (great toe and thumb) of its respective limb. Remember that the upper and lower limbs rotate in opposite directions during their development, resulting in the great toe placed on the medial side of the foot and the thumb on the lateral side of the hand. The alignment of the great saphenous and cephalic veins with the first digits in the limbs denotes them as developmental equivalents. Choice A (Radial vein) is incorrect. The radial vein is a deep vena comitans, a vein that accompanies another structure; in this case, it travels with the radial artery. Like most deep veins of the limbs, this vein actually exists as two veins that travel as companions to the radial artery and are called venae commitantes. Being a deep vein, the radial vein is not an equivalent structure to the subcutaneous great saphenous vein. Choice B (Ulnar vein) is incorrect. The ulnar vein is a deep vena comitans, a vein that accompanies the ulnar artery. The ulnar vein also exists as two venae commitantes that travel collaterally to the ulnar artery within the same connective tissue sheath. The ulnar vein is not an equivalent structure to the great saphenous vein. Choice C (Brachial vein) is incorrect. The brachial vein is a deep, paired venae commitantes that accompanies the brachial artery. This vein is not an equivalent structure to the great saphenous vein due to its deep course within the arm. Choice D (Basilic vein) is incorrect. The basilic vein is a cutaneous vessel that ascends the medial aspect of the arm, in alignment with the fifth digit in the hand. Thus, it is actually on the opposite side of the limb for equivalence with the great saphenous vein. Instead, the basilic vein is the counterpart of the small saphenous vein.

31. In both the upper and lower limbs, the superficial veins begin in a dorsal cutaneous arch that drains into medial and lateral cutaneous veins aligned mainly along the first and fifth digit sides of the limb. Which of the following veins in the upper limb is the equivalent of the great saphenous vein in the lower limb? (A) Radial vein (B) Ulnar vein (C) Brachial vein (D) Basilic vein (E) Cephalic vein

The answer is E: Flexor carpi radialis. The flexor retinaculum (transverse carpal ligament) is a thickening of investing deep fascia on the anterior (ventral) aspect of the wrist that forms the roof of the carpal tunnel. All issues related to the carpal tunnel revolve around an understanding of what structures are contained within the carpal tunnel versus the positions of structures outside the tunnel. The tendons of the flexor carpi radialis and palmaris longus muscles, plus the ulnar nerve and ulnar artery, lie against the superficial aspect of the flexor retinaculum. Any of these structures may be lesioned by a superficial cut across the front (anterior side) of the wrist. Choice A (Flexor digitorum superficialis) is incorrect. The tendons of this muscle are contained within the carpal tunnel, deep to the flexor retinaculum. Choice B (Brachioradialis) is incorrect. This muscle inserts onto the base of the styloid process of the radius. Thus, its tendon is proximal and lateral to the flexor retinaculum, and therefore unrelated to the carpal tunnel. Choice C (Flexor pollicis longus) is incorrect. The tendons of this muscle are contained within the carpal tunnel, deep to the flexor retinaculum. Choice D (Abductor pollicis longus) is incorrect. This muscle is a member of the posterior compartment of the forearm. Its tendon loops out of that compartment to attach onto the lateral side of the base of the first metacarpal (the lateral base of the thumb). Thus, it is well removed from the flexor retinaculum and the carpal tunnel.

33. During an attempted suicide, a depressed young woman slashes the front of her wrist with a razor blade. However, she cuts only to the depth of the superficial aspect of the flexor retinaculum before passing out at the sight of her own blood. Which of the following muscle tendons may be severed? (A) Flexor digitorum superficialis (B) Brachioradialis (C) Flexor pollicis longus (D) Abductor pollicis longus (E) Flexor carpi radialis

The answer is A: Ulnar nerve in the elbow. Situations in which peripheral nerves are compressed or otherwise entrapped where they pass through narrow spaces ("tunnels") in muscles and/or osseo-fascial units are generally referred to as "tunnel syndromes." Such conditions may result in periodic or constant motor and/or sensory deficits. The ulnar nerve crosses the elbow in a narrow space between the olecranon process and the medial epicondyle of the humerus, on the posteromedial (ulnar) side of the joint. The ulnar nerve can be compressed between these bony landmarks or between the humeral and ulnar heads of the attachment of the flexor carpi ulnaris. Compression of the ulnar nerve within these areas leads to "cubital tunnel syndrome." The symptoms are exacerbated during events where flexion of the elbow narrows these passageways and compresses the ulnar nerve at the elbow joint. The patient's cubital tunnel syndrome would explain the paresthesia and numbness on the medial aspect of the hand and the diminished fine motor control of the intrinsic hand muscles. Her condition originated due to the forced flexion of the elbow in the motor vehicle accident, which compressed the ulnar nerve in the cubital tunnel. Choice B (Ulnar nerve in the wrist) is incorrect. The ulnar nerve can be compressed between the pisiform and hook of the hamate at the wrist in a condition termed "ulnar canal syndrome" or "Guyon tunnel syndrome." This entrapment syndrome presents with similar signs and symptoms as seen in this patient. However, the ability to flex the wrist would not be affected. In this patient, trauma to the wrist was not reported. Choice C (Median nerve in the wrist) is incorrect. The median nerve is often compressed at the wrist in "carpal tunnel syndrome" because this nerve travels deep to the transverse carpal ligament (flexor retinaculum of the wrist) within the carpal tunnel. This entrapment syndrome presents with paresthesia of the lateral fingers as well as an inability to oppose the thumb and a wasting of the thenar eminence. These symptoms were not reported in this patient. Choice D (Median nerve in the elbow) is incorrect. The median nerve is not usually compressed by forced flexion of the elbow because it lies loosely on the flexor surface of the joint deep to the bicipital aponeurosis. The median nerve can be compressed in the proximal forearm as it passes between the two heads of the pronator teres. However, the symptoms in this patient were due to forced flexion of the elbow and involve the ulnar nerve. Choice E (Median nerve in the axilla) is incorrect. The median nerve would not be damaged in the axilla without significant trauma. Damage to the median nerve in the axilla would lead to weakness in flexing the wrist, loss of pronation of the forearm, and wasting of the thenar muscles. However, these signs and symptoms were not reported.

36. A 56-year-old woman was stopped at a light when her car was rear-ended by another car. She had her right arm on the steering wheel, and the impact caused forced flexion at her elbow. Several months later, she comes to her physician complaining of numbness and a "pins and needles" sensation in her right little finger when she talks on the phone, rests her head on her right hand at work, or spends most of her day typing at work. She also notices the quality of her typing and her ability to play the violin have diminished. Which nerve is compressed at what location? (A) Ulnar nerve in the elbow (B) Ulnar nerve in the wrist (C) Median nerve in the wrist (D) Median nerve in the elbow (E) Median nerve in the axilla

The answer is C: Brachialis muscle. The cubital fossa is a triangular intermuscular space located anterior to the elbow, comparable to the popliteal fossa in the lower limb. It is an important transition zone between the arm and the forearm, containing major blood vessels and nerves. The roof of the cubital fossa is subcutaneous tissue carrying superficial veins, such as the median cubital vein, which is the most common site for venipuncture in the upper limb. The floor of the fossa is formed by two muscles: brachialis and supinator. Either of these muscles could house the deep abscess caused by repeated needle injections in the cubital fossa. Choice A (Brachioradialis muscle) is incorrect. The brachioradialis muscle forms the lateral wall of the cubital fossa. Choice B (Pronator teres muscle) is incorrect. The pronator teres muscle forms the medial wall of the cubital fossa. Choice D (Head of the radius) is incorrect. The head of the radius lies deep to the muscular floor of the cubital fossa, within the elbow joint complex. Choice E (Olecranon fossa of the humerus) is incorrect. The olecranon fossa is located on the posterior distal aspect of the humerus, well removed from the cubital fossa.

38. Because of repeated bad needle sticks, a heroin addict develops an infected abscess in the floor of the cubital fossa. Which of the following structures is the abscess most likely to invade first? (A) Brachioradialis muscle (B) Pronator teres muscle (C) Brachialis muscle (D) Head of the radius (E) Olecranon fossa of the humerus

The answer is D. Inferior dislocation of the head of the humerus may damage the axillary nerve, which arises from the posterior cord of the brachial plexus, runs through the quad- rangular space accompanied by the posterior humeral circumflex vessels around the sur- gical neck of the humerus, and supplies the deltoid and teres minor, which are lateral rotators of the arm.

39. A 7-year-old boy falls from a tree house and is brought to the emergency department of a local hospital. On examination, he has weakness in rotating his arm laterally because of an injury of a nerve. Which of the following conditions is most likely to cause a loss of this nerve function? (A) Injury to the lateral cord of the brachial plexus (B) Fracture of the anatomic neck of the humerus (C) Knife wound on the teres major muscle (D) Inferior dislocation of the head of the humerus (E) A tumor in the triangular space in the shoulder region

The answer is B. Lymph from the breast drains mainly (75%) to the axillary nodes, more specifically to the anterior (pectoral) nodes.

40. A 49-year-old woman is diagnosed as having a large lump in her right breast. Lymph from the cancerous breast drains primarily into which of the following nodes? (A) Apical nodes (B) Anterior (pectoral) nodes (C) Parasternal (internal thoracic) nodes (D) Supraclavicular nodes (E) Nodes of the anterior abdominal wall

The answer is C. The axillary nerve contains no preganglionic sympathetic general visceral efferent (GVE) fibers, but it contains postganglionic sympathetic GVE fibers. The axillary nerve also contains GSA, GSE, and general visceral afferent (GVA) fibers.

41. A 17-year-old boy fell from his motorcycle and complains of numbness of the lateral part of the arm. Examination reveals that the axillary nerve is severed. Which of the following types of axons is most likely spared? (A) Postganglionic sympathetic axons (B) Somatic afferent axons (C) Preganglionic sympathetic axons (D) General somatic efferent axons (E) General visceral afferent axons

The answer is B: Second part of the axillary artery. The lateral thoracic wall receives significant arterial supply from the branches of the axillary artery. The axillary artery is divided into three parts by the overlying pectoralis minor muscle. The lateral thoracic artery, along with the thoracoacromial trunk, typically branches from the second part of the axillary artery, deep to the pectoralis minor. It descends along the lateral border of the pectoralis minor to supply the lateral aspect of the chest wall, including much of the breast and the serratus anterior muscle. However, this vessel is variable and may originate from other source points. Remember, arteries are often named for the regions they supply, not necessarily for their branching patterns. Choice A (First part of the axillary artery) is incorrect. This segment typically gives rise to one branch: the superior (or supreme) thoracic artery, which has a small distribution to the superolateral chest wall. Choice C (Third part of the axillary artery) is incorrect. This part typically gives rise to three branches: the subscapular artery and the anterior and posterior circumflex humeral arteries. The lateral thoracic and subscapular arteries may originate as a common trunk. Choice D (Third part of the subclavian artery) is incorrect. The subclavian artery is divided into three parts by the anterior scalene muscle. Usually, the third part of the subclavian artery has no branches; however, the dorsal scapular artery may occasionally arise from this location. More importantly, none of the branches of the subclavian artery supply the lateral chest wall. Choice E (First part of the brachial artery) is incorrect. The distribution of the brachial artery is limited to the upper limb; it does not supply the lateral chest wall.

40. The lateral thoracic artery provides the main blood supply to the lateral side of the chest wall, including much of the breast. To deter excessive blood loss during a surgical procedure involving the breast, a surgeon can clamp the lateral thoracic artery near its origin. Which of the following arteries gives rise to this artery? (A) First part of the axillary artery (B) Second part of the axillary artery (C) Third part of the axillary artery (D) Third part of the subclavian artery (E) First part of the brachial artery

The answer is E. The radial artery lies on the floor of the anatomic snuffbox. Other struc- tures are not related to the snuffbox. The tendons of the extensor pollicis longus, extensor pollicis brevis, and abductor pollicis longus muscles form the boundaries of the anatomic snuffbox. The scaphoid and trapezium bones form its floor.

42. A construction worker suffers a destructive injury of the structures related to the anatomic snuffbox. Which of the following structures would most likely be damaged? (A) Triquetral bone (B) Trapezoid bone (C) Extensor indicis tendon (D) Abductor pollicis brevis tendon (E) Radial artery

The answer is C. The subscapularis muscle inserts on the lesser tubercle of the humerus. The supraspinatus, infraspinatus, and teres minor muscles insert on the greater tubercle of the humerus. The coracohumeral ligament attaches to the greater tubercle.

43. A rock climber falls on his shoulder, resulting in a chipping off of the lesser tubercle of the humerus. Which of the following structures would most likely have structural and functional damage? (A) Supraspinatus muscle (B) Infraspinatus muscle (C) Subscapularis muscle (D) Teres minor muscle (E) Coracohumeral ligament

The answer is C. The pectoralis minor inserts on the coracoid process, originates from the second to the fifth ribs, and is innervated by the medial and lateral pectoral nerves that arise from the medial and lateral cords of the brachial plexus. It depresses the shoulder and forms the anterior wall of the axilla. The pectoralis minor has no attachment on the clavicle.

5. A 27-year-old patient presents with an inability to draw the scapula forward and downward because of paralysis of the pectoralis minor. Which of the following would most likely be a cause of his condition? (A) Fracture of the clavicle (B) Injury to the posterior cord of the brachial plexus (C) Fracture of the coracoid process (D) Axillary nerve injury (E) Defects in the posterior wall of the axilla

The answer is D. The intercostobrachial nerve arises from the lateral cutaneous branch of the second intercostal nerve and pierces the intercostal and serratus anterior muscles. It may communicate with the medial brachial cutaneous nerve, and it supplies skin on the medial side of the arm. It contains no skeletal motor fibers but does contain sympathetic postganglionic fibers, which supply sweat glands.

6. A 22-year-old patient received a stab wound in the chest that injured the intercostobrachial nerve. Which of the following conditions results from the described lesion of the nerve? (A) Inability to move the ribs (B) Loss of tactile sensation on the lateral aspect of the arm (C) Absence of sweating on the posterior aspect of the arm (D) Loss of sensory fibers from the second intercostal nerve (E) Damage to the sympathetic preganglionic fibers

The answer is A: Flexor digitorum profundus (FDP). Flexion of the distal interphalangeal joint in digits 2 to 5 is produced by the FDP. The actions of this muscle are being tested in this illustration. Choice B (Extensor indicis) is incorrect. The extensor indicis extends the index finger (digit 2), which enables this finger to extend independent of the other fingers. Because the muscle arises from the distal third of the ulna and the interosseous membrane, it also acts to extend the hand at the wrist. The extensor indicis muscle is not involved in flexion of the distal interphalangeal joint, which is being tested in this patient. Choice C (First lumbrical) is incorrect. The first lumbrical muscle extends the interphalangeal joints of the index (second) finger and flexes the metacarpophalangeal joint of the same finger. The first lumbrical is an intrinsic hand muscle that arises off the tendon of the flexor digitorum profundus and inserts into the extensor expansion of the index finger. This muscle is not involved with flexion of the distal interphalangeal joint. Choice D (First dorsal interosseous) is incorrect. The primary movement of the first dorsal interosseous is abduction of the index finger. However, because it inserts into the extensor expansion, it also extends the interphalangeal joints of the index (second) finger and flexes the metacarpophalangeal joint of the same finger. This muscle is not involved with flexion of the distal interphalangeal joint. Choice E (Flexor digitorum superficialis [FDS]) is incorrect. The FDS acts at the proximal interphalangeal joint in digits 2 to 5 and influences the distal interphalangeal joint by binding the tendons of the FDP. However, when the proximal interphalangeal joint is held in extension, the influence of the FDS is eliminated, allowing testing of only the FDP.

6. As part of a physical examination to evaluate muscle function in the hand, a physician holds the proximal interphalangeal joint of his patient's index finger in the extended position and instructs him to try to flex the distal interphalangeal joint, as shown below. Which of the following muscles is the doctor testing? (A) Flexor digitorum profundus (FDP) (B) Extensor indicis (C) First lumbrical (D) First dorsal interosseous (E) Flexor digitorum superficialis (FDS)

The answer is E. The contents of the cubital fossa from medial to lateral side are the median nerve, the brachial artery, the biceps brachii tendon, and the radial nerve. Thus, the median nerve is damaged. The radial recurrent artery ascends medial to the radial nerve.

10. A 35-year-old man walks in with a stab wound to the most medial side of the proximal portion of the cubital fossa. Which of the following structures would most likely be damaged? (A) Biceps brachii tendon (B) Radial nerve (C) Brachial artery (D) Radial recurrent artery (E) Median nerve

The answer is C. The median nerve innervates the abductor pollicis brevis, opponens pol- licis, and two lateral lumbricals. The ulnar nerve innervates all interossei (palmar and dor- sal), the adductor pollicis, and the two medial lumbricals.

17. A 27-year-old pianist with a known carpal tunnel syndrome experiences difficulty in finger movements. Which of the following intrinsic muscles of her hand is paralyzed? (A) Palmar interossei and adductor pollicis (B) Dorsal interossei and lateral two lumbricals (C) Lateral two lumbricals and opponens pollicis (D) Abductor pollicis brevis and palmar interossei (E) Medial two and lateral two lumbricals

The answer is D. Breast cancer may cause dimpling of the overlying skin because of short- ening of the suspensory (Cooper's) ligaments and inverted or retracted nipple because of pulling on the lactiferous ducts. Polymastia is a condition in which more than two breasts are present.

35. During a breast examination of a 56-year-old woman, the physician found a palpable mass in her breast. Which of the following characteristics of breast cancer and its diagnosis is correct? (A) Elevated nipple (B) Polymastia (C) Shortening of the clavipectoral fascia (D) Dimpling of the overlying skin (E) Enlargement of the breast

The answer is C. The supinator and biceps brachii muscles supinate the forearm. The supinator is innervated by the radial nerve, and the biceps brachii is innervated by the musculocutaneous nerve.

44. A 22-year-old female Macarena dancer fell from the stage and complains of elbow pain and inability to supinate her forearm. Which of the following nerves are most likely injured from this accident? (A) Median and ulnar nerves (B) Axillary and radial nerves (C) Radial and musculocutaneous nerves (D) Ulnar and axillary nerves (E) Musculocutaneous and median nerves

The answer is E. The dorsal and palmar interosseous and lumbrical muscles can flex the metacarpophalangeal joints and extend the interphalangeal joints. The palmar interos- seous muscles adduct the fingers, while the dorsal interosseous muscles abduct the fingers. The flexor digitorum profundus flexes the distal interphalangeal (DIP) joints.

4. A 16-year-old patient has weakness flexing the metacarpophalangeal joint of the ring finger and is unable to adduct the same finger. Which of the following muscles is most likely paralyzed? (A) Flexor digitorum profundus (B) Extensor digitorum (C) Lumbrical (D) Dorsal interosseous (E) Palmar interosseous

The answer is E. The ulnar nerve runs down the medial aspect of the arm and behind the medial epicondyle in a groove, where it is vulnerable to damage by fracture of the medial epicondyle. Other nerves are not in contact with the medial epicondyle.

Questions 48-50: A 21-year-old man injures his right arm in an automobile accident. Radiographic examination reveals a fracture of the medial epicondyle of the humerus. 48. Which of the following nerves is most likely injured as a result of this accident? (A) Axillary (B) Musculocutaneous (C) Radial (D) Median (E) Ulnar

The answer is D. The ulnar nerve innervates the adductor pollicis muscle. The radial nerve innervates the abductor pollicis longus and extensor pollicis brevis muscles, whereas the median nerve innervates the abductor pollicis brevis and opponens pollicis muscles.

Questions 48-50: A 21-year-old man injures his right arm in an automobile accident. Radiographic examination reveals a fracture of the medial epicondyle of the humerus. 49. Which of the following muscles is most likely paralyzed as a result of this accident? (A) Extensor pollicis brevis (B) Abductor pollicis longus (C) Abductor pollicis brevis (D) Adductor pollicis (E) Opponens pollicis

The answer is B. The radial nerve runs in the radial groove on the back of the shaft of the humerus with the profunda brachii artery. The axillary nerve passes around the surgical neck of the humerus. The ulnar nerve passes the back of the medial epicondyle. The mus- culocutaneous and median nerves are not in contact with the bone, but the median nerve can be damaged by supracondylar fracture.

Questions 58-62: A 17-year-old boy is injured in an automobile accident. He has a fracture of the shaft of the humerus. 58. Which of the following nerves is most likely damaged? (A) Axillary nerve (B) Radial nerve (C) Musculocutaneous nerve (D) Median nerve (E) Ulnar nerve

The answer is D. This is the second dorsal interosseous muscle, which abducts the middle finger.

70. The patient is unable to abduct her middle finger because of paralysis of this structure.

The answer is C. A lesion of the upper trunk of the brachial plexus results in a condition called "waiter's tip hand," in which the arm tends to lie in medial rotation because of paralysis of lateral rotators and abductors of the arm. The long thoracic nerve, which arises from the root (C5-C7) of the brachial plexus, innervates the serratus anterior muscle that can elevate the arm above the horizontal. The dorsal scapular nerve, which arises from the root (C5), innervates the rhomboid major. The medial side of the arm receives cutaneous innervation from the medial brachial cutaneous nerve of the medial cord. The adductor pollicis is innervated by the ulnar nerve.

1. A 21-year-old patient has a lesion of the upper trunk of the brachial plexus (Erb- Duchenne paralysis). Which of the following is the most likely diagnosis? (A) Paralysis of the rhomboid major (B) Inability to elevate the arm above the horizontal (C) Arm tending to lie in medial rotation (D) Loss of sensation on the medial side of the arm (E) Inability to adduct the thumb

The answer is C. The opponens pollicis inserts on the first metacarpal. All other intrinsic muscles of the thumb, including the abductor pollicis brevis, the flexor pollicis brevis, and the adductor pollicis muscles, insert on the proximal phalanges.

16. A patient with Bennett's fracture (a fracture of the base of the first metacarpal bone) experiences an impaired thumb movement. Which of the following intrinsic muscles of the thumb is most likely injured? (A) Abductor pollicis brevis (B) Flexor pollicis brevis (superficial head) (C) Opponens pollicis (D) Adductor pollicis (E) Flexor pollicis brevis (deep head)

The answer is A: Median nerve. The median nerve controls pronation through the actions of the pronator teres and pronator quadratus muscles in the anterior compartment of the forearm. It also controls much of flexion of the wrist and lateral digits via the actions of most of the other muscles in that compartment. Choice B (Ulnar nerve) is incorrect. The ulnar nerve controls 11⁄2 muscles in the anterior compartment of the forearm (flexor carpi ulnaris and the ulnar half of the flexor digitorum profundus) and most of the intrinsic muscles of the hand. However, neither of these 11⁄2 forearm muscles produces pronation. Choice C (Superficial branch of the radial nerve) is incorrect. The superficial branch of the radial nerve is entirely cutaneous, carrying sensation from the dorsolateral part of the hand. So, cutting this nerve would not result in weakness in pronation of the forearm or flexion of the wrist. Choice D (Deep branch of the radial nerve) is incorrect. The deep branch of the radial nerve supplies the posterior compartment of the forearm. It influences supination via motor control of the supinator muscle, but not pronation. Choice E (Musculocutaneous nerve) is incorrect. The musculocutaneous nerve supplies the anterior compartment of the arm, and then continues distally via its termination as the lateral cutaneous nerve of the forearm. It contributes significantly to control of supination by its innervation of the biceps brachii muscle, but has no effect on wrist flexion.

1. Physical examination of a 40-year-old man injured in an automobile accident indicates that he has suffered nerve damage affecting his left upper limb. The patient exhibits significant weakness when pronating his left forearm and flexing his left wrist. What nerve is most likely damaged? (A) Median nerve (B) Ulnar nerve (C) Superficial branch of the radial nerve (D) Deep branch of the radial nerve (E) Musculocutaneous nerve

The answer is D: Profunda brachii artery. It is important to recognize where neurovascular structures have close positional relations to each other and to underlying bony structures in order to predict the likely second order functional consequences of damage to the bones. In the given AP X-ray, the midshaft of the humerus is fractured slightly distal to the radial groove. At this point, the profunda brachii vessels (deep vessels of the arm) and the radial nerve emerge from the radial groove in a bundle tightly wrapped against the body of the humerus. A fracture here may readily damage any of these neurovascular structures. Lesion of the vessels may produce swelling in the posterior compartment of the arm and loss of supply to the muscles therein. Lesion of the nerve will result in major motor and sensory deficits in the posterior aspect of the forearm and hand. Choice A (Posterior circumflex humeral artery) is incorrect. This vessel travels in companionship with the axillary nerve around the surgical neck of the humerus. Fractures here are the most common injuries to the proximal end of the humerus, especially in the elderly. Choice B (Ulnar nerve) is incorrect. In the arm, the ulnar nerve travels with the superior ulnar collateral artery. Both lie in contact with the posterior side of the large, projecting medial epicondyle of the humerus as they cross the elbow. Choice C (Axillary nerve) is incorrect. The axillary nerve travels in companionship with the posterior circumflex humeral artery around the surgical neck of the humerus. Fractures here are the most common injuries to the proximal end of the humerus, especially in the elderly. Choice E (Median nerve) is incorrect. This large nerve travels with the brachial vessels down the anterior midline of the arm. The nerve lies close to the distal end of the humerus, where it may be damaged due to fractures of the condyle.

10. The given anteroposterior (AP) X-ray depicts a humeral shaft fracture in a 22-year-old man. Given the location of the fracture, which of the following structures is most likely damaged? (A) Posterior circumflex humeral artery (B) Ulnar nerve (C) Axillary nerve (D) Profunda brachii artery (E) Median nerve

The answer is D. Injury to the median nerve produces the ape hand (a hand with the thumb permanently extended). Injury to the radial nerve results in loss of wrist extension, leading to wrist drop. Damage to the upper trunk of the brachial plexus produces waiter's tip hand. A claw hand and flattening of the hypothenar eminence or atrophy of the hypothenar muscles result from damage to the ulnar nerve.

11. The police bring in a murder suspect who has been in a gunfight with a police officer. The suspect was struck by a bullet in the arm; his median nerve has been damaged. Which of the following symptoms is likely produced by this nerve damage? (A) Waiter's tip hand (B) Claw hand (C) Wrist drop (D) Ape hand (E) Flattening of the hypothenar eminence

The answer is D: Abduction of the index finger. The sensory deficit occurs in the cutaneous territory of the ulnar nerve. This nerve controls most of the intrinsic muscles of the hand, including the first dorsal interosseous muscle, which controls abduction of the index finger. The four dorsal interosseous muscles of the hand are innervated by the deep branch of the ulnar nerve and function to abduct digits 2 to 4. Remember that two fingers (the thumb and little finger) have their own muscles dedicated to abduction. Also, remember the mnemonic "DAB," which stands for Dorsal interossei ABduct the fingers. Thus, the associated motor deficit for the ulnar nerve on this list would be loss of abduction of the index finger. Choice A (Pronation) is incorrect. Pronation is governed by the median nerve, which supplies the pronator teres and pronator quadratus muscles. Choice B (Abduction of the wrist) is incorrect. Abduction of the wrist is controlled by the median nerve (supplying the flexor carpi radialis) and the radial nerve (supplying the extensor carpi radialis longus and brevis). Choice C (Extension of the wrist) is incorrect. Extension of the wrist is produced by the radial nerve acting mainly on the extensor carpi radialis longus and brevis and the extensor carpi ulnaris muscles. Choice E (Flexion of the interphalangeal joints of the index finger) is incorrect. Flexion of the interphalangeal joints of the index finger is controlled by branches of the median nerve, which supply the flexor digitorum superficialis muscle (to flex the proximal interphalangeal joint of the second finger) and the flexor digitorum profundus muscle (to flex the distal interphalangeal joint of the index finger).

12. A 23-year-old medical student complains of loss of sensation in the skin on the medial edge of her left hand, including the entire fifth digit. The associated motor deficit probably involves weakness in which of the following? (A) Pronation (B) Abduction of the wrist (C) Extension of the wrist (D) Abduction of the index finger (E) Flexion of the interphalangeal joints of the index finger

The answer is D. The scaphoid bone forms the floor of the anatomic snuffbox, through which the radial artery passes to enter the palm. The radial artery divides into the prin- ceps pollicis artery and the deep palmar arch.

13. A 14-year-old boy falls on his outstretched hand and has a fracture of the scaphoid bone. The fracture is most likely accompanied by a rupture of which of the following arteries? (A) Brachial artery (B) Ulnar artery (C) Deep palmar arterial arch (D) Radial artery (E) Princeps pollicis artery

The answer is A: Accessory nerve. The accessory nerve (CN XI) traverses the posterior triangle of the neck to reach the deep surface of the trapezius muscle after it innervates the sternocleidomastoid muscle. Its position within the triangle is superficial, and it is at this location that this nerve is vulnerable to injury. Damage to the distal accessory nerve would inhibit elevation of the scapula and lateral rotation of the scapula during abduction greater than 90 degrees. Both of these actions were affected in this patient. Choice B (Axillary nerve) is incorrect. The axillary nerve travels through the quadrangular space to reach the deltoid and teres minor muscles. While a lesion of this nerve would affect abduction of the arm, the axillary nerve is not found in the neck, instead arising distally from the posterior cord of the brachial plexus. Moreover, damage to the axillary nerve would cause a sensory deficit in the upper lateral arm, which was not reported. Choice C (Dorsal scapular nerve) is incorrect. This nerve courses into the upper, medial part of the back and the lower neck to supply the levator scapulae and rhomboid muscles. Paralysis of these muscles would result in weakness in elevation and retraction of the scapula and perhaps wasting of the contour of the back under the trapezius muscle. Choice D (Long thoracic nerve) is incorrect. The long thoracic nerve is a branch of the upper three roots (C5-C7) of the brachial plexus. It descends along the medial wall of the axilla and travels superficial to the serratus anterior muscle, which it innervates. This nerve is coalescing in the deep, most inferior region of the neck, so it would not be injured in the location of the incision. Damage to the long thoracic nerve would lead to "winging of the scapula," when the patient is asked to protract the affected shoulder. Choice E (Thoracodorsal nerve) is incorrect. This nerve runs inferiorly through the axilla to supply the latissimus dorsi muscle. Loss of the nerve would result in weakness in extension and medial rotation of the arm, plus wasting of the posterior axillary fold.

13. A dermatologist performed a biopsy on a suspicious mole on the right side of the posterior neck of a 57-year-old male construction worker. Pathology confirmed a malignant melanoma, so the physician excised a substantial amount of tissue surrounding the mole. After the procedure, the patient experienced difficulty elevating his right shoulder and lifting his right arm over his head. No sensory deficits were seen. What nerve was most likely damaged in this patient? (A) Accessory nerve (B) Axillary nerve (C) Dorsal scapular nerve (D) Long thoracic nerve (E) Thoracodorsal nerve

The answer is C. The axillary nerve passes posteriorly around the surgical neck of the humerus, and the radial nerve lies in the radial groove of the middle of the shaft of the humerus. The ulnar nerve passes behind the medial epicondyle, and the median nerve is vulnerable to injury by supracondylar fracture of the humerus, but these nerves lie close to or in contact with the lower portion of the humerus. The musculocutaneous nerve is not in direct contact with the humerus.

14. A 12-year-old boy walks in; he fell out of a tree and fractured the upper portion of his humerus. Which of the following nerves are intimately related to the humerus and are most likely to be injured by such a fracture? (A) Axillary and musculocutaneous (B) Radial and ulnar (C) Radial and axillary (D) Median and musculocutaneous (E) Median and ulnar

The answer is E: Superficial branch of the radial nerve. The superficial branch of the radial nerve is entirely cutaneous, carrying sensation from the dorsolateral part of the hand from the anatomical snuffbox to the midline of the fourth finger. This nerve is vulnerable as it runs posteriorly between the brachioradialis and extensor carpi radialis longus tendons toward the dorsum of the hand. This nerve was damaged at this location, but the area of sensory loss is less than expected due to the overlap from cutaneous branches of the ulnar and median nerves. Choice A (Dorsal cutaneous branch of the ulnar nerve) is incorrect. The dorsal cutaneous branch of the ulnar nerve passes posterior between the ulna and flexor carpi ulnaris to supply the subcutaneous tissue of the dorsal aspect of the posteromedial aspect of the hand, medial to the midline of the fourth finger. Its medial location and its sensory distribution make it an unlikely choice to be involved with this patient. Choice B (Lateral cutaneous nerve of the forearm) is incorrect. The lateral cutaneous nerve of the forearm is the continuation of the musculocutaneous nerve, which exits the arm between the biceps brachii and brachialis muscles. It supplies the skin on the lateral aspects of the forearm and wrist but would not be responsible for the numbness and paresthesia seen in this patient because the sensory deficit is distal to this nerve's normal distribution. Choice C (Posterior cutaneous nerve of the forearm) is incorrect. The posterior cutaneous nerve of the forearm arises from the radial nerve and passes in close proximity to the lateral intermuscular septum of the arm near the origin of the brachioradialis. As its name implies, it supplies the posterior aspect of the forearm, but it does not extend past the wrist. Therefore, it would not be the nerve damaged in this patient. Choice D (Deep branch of the radial nerve) is incorrect. The deep branch of the radial nerve is entirely motor in its distribution to the posterior muscles of the forearm. Its lack of cutaneous innervation makes this choice easy to eliminate.

15. A 36-year-old man broke a window with his fist to rescue his child from a house fire. The man sustained a laceration to the lateral aspect of his right forearm, but he only showed a sensory deficit (numbness and paresthesia) to the dorsolateral aspect of his hand (as denoted by the shaded area within the given photo). What nerve was most likely damaged? (A) Dorsal cutaneous branch of the ulnar nerve (B) Lateral cutaneous nerve of the forearm (C) Posterior cutaneous nerve of the forearm (D) Deep branch of the radial nerve (E) Superficial branch of the radial nerve

The answer is D: Intercostobrachial nerve. The intercostobrachial nerve is the lateral cutaneous branch of the second intercostal nerve. As its name implies, it leaves the thorax by exiting between the second and third ribs (between the ribs = intercosto-) to supply cutaneous innervation to the axilla and medial aspect of the arm (brachium). In some instances, it may also supply skin distal to the elbow. The anesthetic solution would block all of the distal branches of the brachial plexus residing within the axillary sheath, thus sparing the intercostobrachial nerve. Choice A (Long thoracic nerve) is incorrect. The long thoracic nerve does not have a cutaneous distribution and provides only motor innervation to the serratus anterior. Because the long thoracic nerve arises from the ventral rami of C5-C7, it would not be affected by the anesthetic solution, especially if an occlusive tourniquet was utilized to retain the solution. Such a procedure is used to prevent spread of the anesthetic into the neck where it could affect the phrenic nerve and negatively affect respiration. Choice B (Median nerve) is incorrect. The median nerve does not branch proximal to the elbow, and its sensory distribution is limited to distal to the wrist. Furthermore, this nerve would be blocked by effective administration of the anesthetic reagent. Choice C (Medial cutaneous nerve of the arm) is incorrect. The medial cutaneous nerve of the arm arises from the medial cord of the brachial plexus, and it would be blocked by effective administration of the anesthetic solution into the axillary sheath. This nerve would also provide sensation to the area (medial aspect of the arm) that remains sensitive to pain, but it should be blocked by the drug. Choice E (Ulnar nerve) is incorrect. The ulnar nerve does not branch proximal to the elbow, and its sensory distribution is limited to distal to the wrist. Furthermore, this nerve would be blocked by effective administration of the anesthetic solution.

16. An anesthesiologist administers an anesthetic solution into the axillary sheath of a 19-year-old college baseball player in preparation for repair of the ulnar collateral ligament of the elbow. After 5 minutes, the patient experiences numbness and paresthesia distal to the middle aspect of the arm; however, the medial aspect of the arm and elbow remain sensitive to pain. What nerve provides sensory innervation to the sensitive area and was not blocked by the anesthetic solution? (A) Long thoracic nerve (B) Median nerve (C) Medial cutaneous nerve of the arm (D) Intercostobrachial nerve (E) Ulnar nerve

The answer is A: Axillary nerve. The axillary nerve may be damaged in approximately one of seven shoulder dislocations. This nerve innervates the deltoid and teres minor muscles as well as supplying innervation to the skin overlying the deltoid in the superolateral aspect of the arm. Loss of innervation to the deltoid muscle would explain the weakness in abduction of the upper limb. The teres minor assists the Infraspinatus muscle in external rotation of the shoulder. Choice B (Median nerve) is incorrect. The median nerve does not branch proximal to the elbow, and its sensory distribution is limited to the hand. The median nerve could not be responsible for the patient's motor and sensory deficits. Choice C (Ulnar nerve) is incorrect. The ulnar nerve can be damaged during shoulder dislocations; however, its sensory distribution is limited to distal to the wrist, and the first muscle it innervates is in the forearm (flexor carpi ulnaris). Therefore, the ulnar nerve could not be responsible for this patient's motor and sensory deficits. Choice D (Radial nerve) is incorrect. The radial nerve supplies motor innervation to the posterior compartments of the arm and forearm. Damage to this nerve would cause weakness in extending at the elbow and wrist joints. The radial nerve gives rise to the posterior cutaneous nerves of the arm and forearm as well as the inferior lateral cutaneous nerve of the arm. However, it would not affect the superior lateral cutaneous nerve of the arm, arising from the axillary nerve, which was damaged in this patient. The radial nerve would also not affect abduction and external rotation of the shoulder. Choice E (Musculocutaneous nerve) is incorrect. The musculocutaneous nerve can be damaged during shoulder dislocations; however, this nerve supplies the motor innervation to the anterior compartment of the arm. Trauma to this nerve would lead to weakness in flexing the elbow and supinating the forearm. Its sensory distribution is limited to the lateral aspect of the forearm, so it was not the nerve damaged in this patient.

17. A 17-year-old male football player suffers a shoulder injury and arrives at the ER 2 hours after the injury. The physician diagnoses a shoulder dislocation, and after administration of a local anesthetic solution, the doctor repositions the head of the humerus into the glenoid cavity of the scapula (reduction). No fractures are seen on X-rays. However, the patient displays weakness in abduction and external rotation at the shoulder. A loss of sensation is also noted at the superior and lateral aspects of the arm. What nerve was most likely damaged in this injury? (A) Axillary nerve (B) Median nerve (C) Ulnar nerve (D) Radial nerve (E) Musculocutaneous nerve

The answer is C. The ulnar bursa, or common synovial flexor sheath, contains the tendons of both the flexor digitorum superficialis and profundus muscles. The radial bursa envel- ops the tendon of the flexor pollicis longus. The tendons of the flexor carpi ulnaris and the palmaris longus are not contained in the ulnar bursa.

18. A 31-year-old roofer walks in with tenosynovitis resulting from a deep penetrated wound in the palm by a big nail. Examination indicates that he has an infection in the ulnar bursa. This infection most likely resulted in necrosis of which of the following tendons? (A) Tendon of the flexor carpi ulnaris (B) Tendon of the flexor pollicis longus (C) Tendon of the flexor digitorum profundus (D) Tendon of the flexor carpi radialis (E) Tendon of the palmaris longus

The answer is E: Muculocutaneous nerve. A positive response to the myotatic biceps reflex confirms the integrity of the musculocutaneous nerve and the C5 and C6 spinal segments, from which this nerve arises. The musculocutaneous nerve supplies motor innervation and proprioception to the muscles in the anterior compartment of the arm, including the coracobrachialis, biceps brachii (tested here), and brachialis. Lesioning the musculocutaneous nerve would lead to loss of proprioception and weakness in flexing the elbow (via the biceps brachii and brachialis muscles) and supinating the forearm (via the biceps brachii), resulting in a negative myotatic biceps reflex. Choice A (Axillary nerve) is incorrect. The axillary nerve innervates the deltoid and teres minor muscles as well as the skin overlying the deltoid in the superolateral aspect of the arm. Loss of innervation to the deltoid muscle would cause weakness in abduction of the upper limb. The teres minor and posterior head of the deltoid are responsible for external rotation of the shoulder. Choice B (Median nerve) is incorrect. The median nerve innervates 6.5 of the 8 anterior forearm muscles. This nerve would help with flexion of the forearm at the elbow; however, the myotatic biceps reflex test specifically tests the innervation of the biceps brachii muscle, which is innervated by the musculocutaneous nerve. Choice C (Ulnar nerve) is incorrect. The ulnar nerve innervates only 1.5 muscles of the forearm, specifically the flexor carpi ulnaris and the ulnar side of the flexor digitorum profundus. This nerve also innervates most of the intrinsic muscles of the hand. It is not involved with the myotatic biceps reflex test. Choice D (Radial nerve) is incorrect. The radial nerve supplies motor innervation to the posterior compartments of the arm and forearm. Damage to this nerve would cause weakness in extension at the elbow and wrist joints. The radial nerve is responsible for the posterior cutaneous nerves of the arm and forearm as well as the inferior lateral cutaneous nerve of the arm. However, it is responsible for extension at the elbow, not the flexion of the elbow tested with the myotatic biceps reflex test.

18. A physician tests the myotatic biceps reflex as shown. A normal response of involuntary contraction of the biceps brachii muscle is noted. This reflex confirms the integrity of what nerve? (A) Axillary nerve (B) Median nerve (C) Ulnar nerve (D) Radial nerve (E) Musculocutaneous nerve

The answer is D: Radial nerve. This case represents a classic presentation of "Saturday Night Palsy," where the radial nerve is compressed against the humerus in the arm. Remember, the radial nerve supplies motor innervation to the posterior compartments of the arm and forearm, so damage to this nerve would cause weakness in extending the elbow and wrist. This patient is unable to extend the wrist when the hand is placed in a pronated position ("wrist drop"), implying damage to the radial nerve. Moreover, the superficial branch of the radial nerve is responsible for cutaneous innervation over much of the dorsum of the hand, which explains the numbness and paresthesia in his hand. Choice A (Axillary nerve) is incorrect. The axillary nerve innervates the deltoid and teres minor muscles as well as the skin overlying the deltoid in the superolateral aspect of the arm. Compressing the axillary nerve would not affect wrist function or cause paresthesia distal to the wrist, which was seen in this student. Choice B (Median nerve) is incorrect. The median nerve is responsible for flexion at the wrist; however, this patient is having trouble with wrist extension. The sensory distribution for the median nerve is primarily on the palmar side of the hand rather than the dorsum of the hand, which is seen in this patient. Choice C (Ulnar nerve) is incorrect. The ulnar nerve is responsible for sensory innervation on the dorsum of the hand that is limited to the ulnar side (medial to the midline of the fourth finger). Damage to the ulnar nerve would not result in "wrist drop," which is an indication of a radial nerve injury. The ulnar nerve does supply motor innervation to most of the intrinsic muscles of the hand. Choice E (Musculocutaneous nerve) is incorrect. The musculocutaneous nerve supplies motor innervation to the anterior compartment of the arm and cutaneous innervation to the lateral aspect of the forearm. Therefore, damage to this nerve would not affect wrist function.

19. A 21-year-old male college student reports to the student health clinic on Monday morning, the day after the Super Bowl. He explains that he was intoxicated and lost consciousness with his upper limbs draped over the back of a couch. He complains of numbness and paresthesia over the dorsum of his hand on the radial side and is unable to support the weight of his left hand when the hand is placed in a pronated position (see photo). What nerve was most likely damaged in this individual? (A) Axillary nerve (B) Median nerve (C) Ulnar nerve (D) Radial nerve (E) Musculocutaneous nerve

The answer is A. The abductors of the arm are the deltoid and supraspinatus muscles, which are innervated by the axillary and suprascapular nerves, respectively. The thoraco- dorsal nerve supplies the latissimus dorsi, which can adduct, extend, and rotate the arm medially. The upper and lower subscapular nerves supply the subscapularis, and the lower subscapular nerve also supplies the teres major; both of these structures can adduct and rotate the arm medially. The musculocutaneous nerve supplies the flexors of the arm, and the radial nerve supplies the extensors of the arm. The dorsal scapular nerve supplies the levator scapulae and rhomboid muscles; these muscles elevate and adduct the scapula, respectively.

19. An 18-year-old boy involved in an automobile accident presents with arm that cannot abduct. His paralysis is caused by damage to which of the following nerves? (A) Suprascapular and axillary (B) Thoracodorsal and upper subscapular (C) Axillary and musculocutaneous (D) Radial and lower subscapular (E) Suprascapular and dorsal scapular

The answer is A: Upper trunk of the brachial plexus. The illustration shows an injury in which the cervicobrachial angle (the angle between the neck and shoulder) is stretched widely. This abnormal impact eventually results in the postural presentation of a "waiter's tip" deformity (Erb-Duchenne palsy). This combination of injury and postural deformity is related to damage to both the C5 and C6 roots or upper trunk of the brachial plexus. The C5 and C6 roots converge to form the upper trunk of the brachial plexus and contribute heavily to the suprascapular, axillary, and musculocutaneous nerves. The suprascapular nerve supplies the supraspinatus and infraspinatus muscles. The axillary nerve controls the deltoid and teres minor muscles. The musculocutaneous nerve supplies the anterior compartment of the arm (coracobrachialis, biceps brachii, brachialis muscles). Therefore, a significant weakness in abduction and lateral rotation of the shoulder, flexion of the shoulder and elbow, and supination of the forearm would result from this brachial plexus injury. The ultimate postural deformity is a contracture effect in which the intact muscles act unopposed to draw the limb into a position that is the opposite of the actions of the affected muscles. Choice B (Lower roots of the brachial plexus) is incorrect. The lower roots (C8, T1) have a strong projection into the ulnar nerve. Trauma here would result in an ultimate postural deformity of "claw hand" due to the loss of flexion of the medial digits. That injury does not match this patient's clinical presentation. Choice C (Posterior divisions of the brachial plexus) is incorrect. The posterior divisions supply the radial, axillary, upper and lower subscapular, and thoracodorsal nerves. The primary postural effect resulting from trauma here would be a case of "wrist drop" expressed due to loss of the extensor muscles innervated by the radial nerve. Choice D (Medial cord of the brachial plexus) is incorrect. The medial cord projects into the ulnar and median nerves. Damage to the ulnar nerve will result in "claw hand." The classic median nerve deformity of "ape hand" probably would not be realized because the median nerve also receives a strong input from the lateral cord. Choice E (Lateral root of the median nerve) is incorrect. The median nerve is formed by lateral and medial roots derived from the lateral and medial cords, respectively. Trauma to either root would weaken the territory of the median nerve but likely not result in the classic median nerve postural defect of "ape hand." The lateral root does not contribute to the suprascapular and axillary nerves, which have been affected in this patient.

2. A 50-year old man falls off a ladder while cleaning his windows, landing on the ground as seen in the given drawing. He does not seek medical aid, believing his general soreness will go away with time. However, after several months, he develops a postural deformity of his left upper limb that includes an adducted, medially rotated, and extended shoulder, extended elbow, and pronated forearm. The injury and subsequent condition reflect damage to what structure? (A) Upper trunk of the brachial plexus (B) Lower roots of the brachial plexus (C) Posterior divisions of the brachial plexus (D) Medial cord of the brachial plexus (E) Lateral root of the median nerve

The answer is C. The posterior humeral circumflex artery anastomoses with an ascending branch of the profunda brachii artery, whereas the lateral thoracic and subscapular arter- ies do not. The superior ulnar collateral and radial recurrent arteries arise inferior to the origin of the profunda brachii artery.

20. A 17-year-old boy with a stab wound received multiple injuries on the upper part of the arm and required surgery. If the brachial artery were ligated at its origin, which of the following arteries would supply blood to the profunda brachii artery? (A) Lateral thoracic (B) Subscapular (C) Posterior humeral circumflex (D) Superior ulnar collateral (E) Radial recurrent

The answer is B: Lower trunk of the brachial plexus. This woman has experienced a lower brachial plexus injury due to forced abduction of the upper limb during the accident. This injury presents with numbness and paresthesia in the C8 and T1 dermatomes, which supply the axilla and medial aspect of her upper limb. These nerve roots primarily supply the medial cord of the brachial plexus, which creates the ulnar nerve. Due to damage to the ulnar nerve, she is experiencing weakness in the movement of her left hand. The abduction and adduction of the fingers are controlled by the deep branch of the ulnar nerve by supplying the dorsal interosseous and palmar interosseous muscles, respectively. Choice A (Upper trunk of the brachial plexus) is incorrect. Damage to the upper trunk of the brachial plexus results in Erb-Duchenne palsy ("waiter's tip malformation"). In this injury, the patient presents with significant weakness in abduction and lateral rotation of the shoulder, flexion of the shoulder and elbow, and supination of the forearm. Choice C (Posterior cord of the brachial plexus) is incorrect. The posterior cord of the brachial plexus gives rise to the axillary and radial nerves. Damage to the axillary nerve causes weakness in abduction of the shoulder due to loss of the deltoid muscle. Damage to the radial nerve results in inability to extend at the elbow or wrist (leading to "wrist drop"). However, this patient had significant weakness in the movements of the hand, which implies damage to the contributions of the ulnar nerve. Choice D (Lateral cord of the brachial plexus) is incorrect. Damage to the lateral cord of the brachial plexus results in signs and symptoms similar to those seen in Erb-Duchenne palsy, or the "waiter's tip malformation." In this injury, the patient presents with significant weakness in abduction and lateral rotation of the shoulder, flexion of the shoulder and elbow, and supination of the forearm. Choice E (Long thoracic nerve) is incorrect. A lesion of the long thoracic nerve leads to a "winged scapula" due to the subsequent paralysis of the serratus anterior muscle. Additionally, the affected arm cannot be abducted above the horizontal plane because the serratus anterior is not available to superiorly rotate the glenoid cavity of the scapula to allow full abduction. This deficit was not seen in this patient.

20. A 50-year female equestrian is thrown from a startled horse and dragged by the reins, which were wrapped around her left wrist, for some distance. At the ER, she is experiencing pain and paresthesia in the axilla and medial aspect of her upper limb. Despite being left-handed, she has marked weakness in the movements of her dominant hand, especially abduction and adduction of the fingers. What structure was most likely damaged in this woman? (A) Upper trunk of the brachial plexus (B) Lower trunk of the brachial plexus (C) Posterior cord of the brachial plexus (D) Lateral cord of the brachial plexus (E) Long thoracic nerve

The answer is D. The metacarpophalangeal joint of the ring finger is flexed by the lumbri- cal, palmar, and dorsal interosseous muscles, which are innervated by the ulnar nerve. The extensor digitorum, which is innervated by the radial nerve, extends this joint. The muscu- locutaneous and axillary nerves do not supply muscles of the hand. The median nerve supplies the lateral two lumbricals, which can flex metacarpophalangeal joints of the index and middle fingers.

21. A 23-year-old woman who receives a deep cut to her ring finger by a kitchen knife is unable to move the metacarpophalangeal joint. Which of the following pairs of nerves was damaged? (A) Median and ulnar (B) Radial and median (C) Musculocutaneous and ulnar (D) Ulnar and radial (E) Radial and axillary

The answer is D: Meromelia. Thalidomide was once used widely to help combat morning sickness during pregnancy. Following a wave of children born with limb malformations, the medication was discovered to be a significant teratogen and was taken off the market. Currently, thalidomide is being used to treat AIDS and cancer patients, raising concerns of a new rash of children born with limb defects. The specific malformation in this case is termed meromelia, a partial absence of limb segments. Meromelia is a type of reduction defect, in which whole or partial limb components are absent. Because of the resemblance of the limbs to a seal's flippers, this defect is also termed phocomelia ("seal limb"). Choice A (A dysplasia) is incorrect. Dysplasias are malformations in which elements are malformed. In this case, limb components are missing rather than malformed. Choice B (A duplication defect) is incorrect. Duplication defects are characterized by the presence of supernumerary (extra) elements. An example is polydactyly, in which extra digits are present. Choice C (Micromelia) is incorrect. Micromelia ("small limb") is a type of dysplasia. In this, the limb segments are present but are abnormally short. Choice E (Amelia) is incorrect. Amelia ("without limb") is a type of reduction defect in which an entire extremity is absent.

21. A child is born to a young woman who had utilized thalidomide to help relieve her morning sickness early in her pregnancy. The infant suffers the congenital defects shown here. She is missing the proximal segments of both upper and lower limbs. The hands and feet that are present are attached to the trunk of the body and resemble small seal's flippers. Which of the following is the correct term for this malformation? (A) Adysplasia (B) A duplication defect (C) Micromelia (D) Meromelia (E) Amelia

The answer is B: Latissimus dorsi. The axilla is a large, pyramidal space between the side of the chest and the upper part of the brachium. Its major importance is as a passageway from the root of the neck to the upper limb. The axilla is demarcated by four walls: anterior, posterior, medial, lateral. The posterior wall is composed of the latissimus dorsi, teres major, and subscapularis muscles. The posterior axillary fold forms the palpable lower margin of the wall and is composed of the latissimus dorsi and teres major. The subscapularis is not part of the posterior axillary fold. Damage to the latissimus dorsi would severely hinder adduction, extension, and medial rotation of the arm. Choice A (Pectoralis minor) is incorrect. The pectoralis minor contributes to the formation of the anterior wall of the axilla. The pectoralis major forms the bulk of the anterior wall and creates the noticeable anterior axillary fold. Choice C (Levator scapulae) is incorrect. The levator scapulae muscle passes out of the neck to attach onto the superior angle of the scapula. It is far removed from any of the walls of the axilla and does not contribute to rotation and adduction of the arm. Choice D (Serratus anterior) is incorrect. The serratus anterior lies against the thoracic wall and forms much of the medial wall of the axilla along with the thoracic wall. It fixes the scapula to the thoracic wall and has no affect on rotation of the arm. Choice E (Teres minor) is incorrect. The teres minor is located immediately above the teres major. However, it is not a component of the posterior wall of the axilla and normally contributes to lateral rotation of the arm.

23. A 65-year-old man is brought to the emergency room after being attacked in his office by a disgruntled co-worker. The attacker reportedly used a long, narrow-bladed letter-opener to inflict multiple stab wounds to the man's back. Physical examination shows a puncture wound in the posterior axillary fold. The patient presents with weakness in extension, adduction, and medial rotation of his arm. Which of the following muscles is most likely cut in this injury? (A) Pectoralis minor (B) Latissimus dorsi (C) Levator scapulae (D) Serratus anterior (E) Teres minor

The answer is D. The supinator and biceps brachii muscles, which are innervated by the radial and musculocutaneous nerves, respectively, produce supination of the forearm. This is a question of two muscles that can supinate the forearm.

24. A 38-year-old homebuilder was involved in an accident and is unable to supinate his forearm. Which of the following nerves are most likely damaged? (A) Suprascapular and axillary (B) Musculocutaneous and median (C) Axillary and radial (D) Radial and musculocutaneous (E) Median and ulnar

The answer is D: Axillary nerve. The quadrangular space of the shoulder is an important passageway allowing the posterior humeral circumflex vessels and their companion axillary nerve to pass from the axilla to the posterior aspect of the shoulder. The neurovascular bundle runs across the surgical neck of the humerus to enter the quadrangular space. The space itself (sometimes termed the lateral axillary hiatus) is formed by four structures: teres major, teres minor, long head of the triceps, surgical neck of the humerus. The vessels contribute to the collateral network around the shoulder. The axillary nerve supplies the teres minor and deltoid muscles and a cutaneous area on the superolateral aspect of the arm (i.e., the skin overlying the lower aspect of the deltoid muscle). Choice A (Musculocutaneous nerve) is incorrect. The musculocutaneous nerve is a terminal branch of the lateral cord of the brachial plexus within the axilla. It supplies the anterior compartment of the arm and is not related to the quadrangular space. Choice B (Lateral cord of the brachial plexus) is incorrect. The lateral cord occupies a relatively lateral position in the axilla but is not related to the quadrangular space. The axillary nerve originates from the posterior cord of the brachial plexus. Choice C (Radial nerve) is incorrect. The radial and axillary nerves are the terminal branches of the posterior cord of the brachial plexus. The radial nerve runs through the radial (spiral) groove in the midshaft of the humerus to emerge through the lower triangular space, just below the quadrangular space, and enter the posterior compartment of the arm. Choice E (Medial cutaneous nerve of the arm) is incorrect. The medial cutaneous nerve of the arm is a branch of the medial cord of the brachial plexus. It is far removed from the quadrangular space.

24. Physical examination of a 45-year-old man who had been stabbed in the back of the shoulder shows a deep wound penetrating into the quadrangular space of the shoulder, causing bleeding from the severed blood vessels there. Which of the following neural structures is most likely damaged as well? (A) Musculocutaneous nerve (B) Lateral cord of the brachial plexus (C) Radial nerve (D) Axillary nerve (E) Medial cutaneous nerve of the arm

The answer is D. The ulnar nerve supplies sensory fibers to the skin over the palmar and dorsal surfaces of the medial third of the hand and the medial one and one-half fingers. The median nerve innervates the skin of the lateral side of the palm; the palmar side of the lateral three and one-half fingers; and the dorsal side of the index finger, the middle finger, and one-half of the ring finger. The radial nerve innervates the skin of the radial side of the hand and the radial two and one-half digits over the proximal phalanx.

25. A 31-year-old patient complains of sensory loss over the anterior and posterior surfaces of the medial third of the hand and the medial one and one-half fingers. He is diagnosed by a physician as having "funny bone" symptoms. Which of the following nerves is injured? (A) Axillary (B) Radial (C) Median (D) Ulnar (E) Musculocutaneous

The answer is E: The second part of the axillary artery lies deep to the muscle. The pectoralis minor muscle overlies the axillary artery in such a way as to divide it into three parts: first (prepectoral; medial), second (subpectoral; deep), third (postpectoral; lateral). The first part is medial to the pectoralis minor, running from the lateral border of the first rib to the medial border of the pectoralis minor. The second part is deep to the muscle. The third part is lateral to the pectoralis minor, running from the lateral border of the muscle to the inferior border of the teres major muscle. Choice A (The lateral cord of the brachial plexus lies lateral to the muscle) is incorrect. The cords of the brachial plexus are so named by their positions relative to the second part of the axillary artery. Choice B (The clavipectoral triangle lies lateral to the muscle) is incorrect. The clavipectoral triangle lies medial to the pectoralis minor, between the muscle and the clavicle. This fascia-roofed space is noteworthy because it is pierced by the lateral pectoral nerve (on its way to the clavicular head of the pectoralis major), branches of the thoracoacromial artery, and the cephalic vein (on its way to empty into the axillary vein). Choice C (The anterior axillary lymph nodes lie along the medial border of the muscle) is incorrect. The anterior (pectoral; level 1) group of axillary lymph nodes lie along the lateral border of the pectoralis minor. Choice D (The lateral wall of the axillary fossa includes the muscle) is incorrect. The pectoralis minor forms the anterior wall of the axillary fossa, along with the pectoralis major.

25. The pectoralis minor muscle is an important landmark in identifying and describing neighboring structures in the chest and axillary regions. Which of the following relationships of the pectoralis minor is correct? (A) The lateral cord of the brachial plexus lies lateral to the muscle (B) The clavipectoral triangle lies lateral to the muscle (C) The anterior axillary lymph nodes lie along the medial border of the muscle (D) The lateral wall of the axillary fossa includes the muscle (E) The second part of the axillary artery lies deep to the muscle

The answer is C. The anterior interosseous nerve is a branch of the median nerve and sup- plies the flexor pollicis longus, half of the flexor digitorum profundus, and the pronator quadratus. The median nerve supplies the pronator teres, flexor digitorum superficialis, palmaris longus, and flexor carpi radialis muscles. A muscular branch (the recurrent branch) of the median nerve innervates the thenar muscles.

26. A patient with a deep stab wound in the middle of the forearm has impaired movement of the thumb. Examination indicates a lesion of the anterior interosseous nerve. Which of the following muscles is paralyzed? (A) Flexor pollicis longus and brevis (B) Flexor pollicis longus and opponens pollicis (C) Flexor digitorum profundus and pronator quadratus (D) Flexor digitorum profundus and superficialis (E) Flexor pollicis brevis and pronator quadratus

The answer is C. The flexor digitorum profundus muscle flexes the DIP joints of the index and middle fingers and is innervated by the median nerve, which also supplies sensation over the distal part of the second digit. The same muscle flexes the DIP joints of the ring and little fingers but receives innervation from the ulnar nerve, which also innervates the hypothenar muscles. The median nerve innervates the thenar muscles. The radial nerve innervates the supinator, abductor pollicis longus, and extensor pollicis longus and brevis muscles. The ulnar nerve innervates the adductor pollicis. The musculocutaneous nerve supplies the biceps brachii that can supinate the arm.

27. A 29-year-old patient comes in; he cannot flex the distal interphalangeal (DIP) joint of the index finger. His physician determines that he has nerve damage from a supracondylar fracture. Which of the following conditions is also a symptom of this nerve damage? (A) Inability to flex the DIP joint of the ring finger (B) Atrophy of the hypothenar eminence (C) Loss of sensation over the distal part of the second digit (D) Paralysis of all the thumb muscles (E) Loss of supination

"The answer is B: Median nerve. Each of the five terminal branches of the brachial plexus (musculocutaneous, median, ulnar, radial, and axillary nerves) passes through a muscular or osseofascial tunnel at some point in its distribution, where it may be subject to entrapment in a tunnel syndrome. The pronator teres muscle arises via two heads, one from the medial epicondyle of the humerus and the other from the coronoid process of the ulna, with a tendinous arch connecting them. The median nerve exits the cubital fossa and enters the forearm by passing between these heads, where it may be unduly compressed in a pronator teres syndrome. This condition would influence much of the median nerve territory in the forearm plus the entire median nerve territory in the hand. Choice A (Deep branch of the radial nerve) is incorrect. The radial nerve descends from the arm into the cubital fossa, where it divides into superficial and deep branches. The deep branch of the radial nerve pierces the supinator muscle, winds around the proximal end of the radius within the substance of that muscle, and passes into the deep posterior compartment of the forearm as the posterior interosseous nerve. The nerve may be entrapped within the supinator, resulting in a supinator syndrome. Such a condition would affect the deeper, more distal extensor muscles arising in the forearm and some sensory areas in the wrist joints. Choice C (Deep branch of the ulnar nerve) is incorrect. The ulnar nerve enters the hand superficial to the flexor retinaculum, runs through a groove between the pisiform and hook of the hamate (Guyon canal), and divides into superficial and deep branches at the base of the hypothenar eminence. The deep branch curls deeply there and enters the deep lying adductor/interosseous compartment in the palmar aspect of the hand. Compression of the ulnar nerve in Guyon canal may cause a Guyon tunnel syndrome, which affects the hypothenar muscles, medial two lumbricals, all interossei, adductor pollicis, and a large sensory area on both palmar and dorsal sides of the hand. Choice D (Superficial branch of the ulnar nerve) is incorrect. The superficial branch of the ulnar nerve does not enter a tunnel and is not subject to a tunnel syndrome. This nerve supplies the palmaris brevis muscle but is mostly cutaneous across the palmar and dorsal aspects of the medial third of the hand. Choice E (Musculocutaneous nerve) is incorrect. This nerve penetrates the coracobrachialis muscle, supplies the three flexor muscles in the anterior compartment of the arm, and continues into the forearm as the lateral cutaneous nerve of the forearm. Entrapment of the nerve within the coracobrachialis is rare."

28. "Pronator teres syndrome" is a condition in which one of the following nerves is excessively compressed where it passes between the two heads of the pronator teres muscle. Which of the following nerves is entrapped? (A) Deep branch of radial nerve (B) Median Nerve (C) Deep branch of ulnar nerve (D) Superficial branch of ulnar nerve (E) Musculocutaneous nerve

The answer is C. The carpal tunnel contains the median nerve and the tendons of flexor pollicis longus, flexor digitorum profundus, and flexor digitorum superficialis muscles. Carpal tunnel syndrome results from injury to the median nerve, which supplies the the- nar muscle. Thus, injury to this nerve causes the flattened thenar eminence. The middle finger has no attachment for the adductors. The ulnar nerve innervates the medial half of the flexor digitorum profundus muscle, which allows flexion of the DIP joints of the ring and little fingers. The ulnar nerve supplies the skin over the medial one and one-half fin- gers and adductor pollicis muscle.

29. A secretary comes in to your office complaining of pain in her wrists from typing all day. You determine that she likely has carpal tunnel syndrome. Which of the following conditions would help you determine the diagnosis? (A) Inability to adduct the little finger (B) Inability to flex the DIP joint of the ring finger (C) Flattened thenar eminence (D) Loss of skin sensation of the medial one and one-half fingers (E) Atrophied adductor pollicis muscle

The answer is B: Scaphoid. The most commonly fractured carpal bone is the scaphoid that forms the floor of the anatomical snuffbox. This area is a fossa located between the three long tendons of the thumb (tendons of abductor pollicis longus and extensor pollicis brevis laterally and tendon of the extensor pollicis longus medially). The scaphoid bone is frequently broken when an individual falls with an outstretched hand and lands on the palm with the hand abducted. A broken scaphoid bone is commonly seen in individuals under the age of 30. The scaphoid is broken due to its unfortunate position between the downward force transmitted by the weight of the upper limb and the upward force due to the impact of hitting the floor. The proximal aspect of a broken scaphoid bone can suffer from avascular necrosis due to its blood supply entering the bone distally. Choice A (Capitate) is incorrect. The capitate is located centrally in the distal row of carpal bones and articulates with most of the carpal bones (the triquetrum, pisiform, and trapezium being the exceptions). This bone is not located in the floor of the anatomical snuffbox, so it was not the most likely bone injured in this patient. Choice C (Hamate) is incorrect. The hamate is located in the distal row of carpal bones on the ulnar side. It is not related to the anatomical snuffbox, so it would not produce pain in the base of the thumb when broken. Choice D (Trapezium) is incorrect. The trapezium is located at the base of the thumb, but it is not commonly fractured when a person falls with an outstretched hand. A mnemonic for the position of the trapezium is "trapezium articulated with the thumb." Choice E (Pisiform) is incorrect. The pisiform is located in the proximal carpal row and is a sesamoid bone located within the tendon of the flexor carpi ulnaris muscle. Its position on the ulnar side of the wrist would not cause pain in the anatomical snuffbox.

3. A 21-year-old man goes to his college campus health clinic complaining of soreness in his left wrist after falling on an outstretched hand during a basketball game the previous day. He is supporting his left wrist and indicates that the pain worsens with movement and is minimized with inactivity. There is no loss of feeling in his hand, nor does he have trouble grasping or holding objects. The physician exacerbates the wrist pain by applying pressure to the base of the thumb in the anatomical snuffbox (see photo). Radiographic imaging will confirm a break of which carpal bone? (A) Capitate (B) Scaphoid (C) Hamate (D) Trapezium (E) Pisiform

The answer is C. The rotator cuff consists of the tendons of the supraspinatus, infraspina- tus, subscapularis, and teres minor muscles. It stabilizes the shoulder joint by holding the head of the humerus in the glenoid cavity during movement. The teres major inserts on the medial lip of the intertubercular groove of the humerus.

32. A patient has a torn rotator cuff of the shoulder joint as the result of an automobile accident. Which of the following muscle tendons is intact and has normal function? (A) Supraspinatus (B) Subscapularis (C) Teres major (D) Teres minor (E) Infraspinatus

The answer is A: Posterior circumflex humeral artery. The radiograph shows a fracture of the surgical neck of the humerus. The posterior humeral circumflex artery, accompanied by the axillary nerve, lies against the posterior aspect of the surgical neck as it passes into the quadrangular space of the shoulder. This fracture places both of these structures in immediate danger. Choice B (Brachial artery) is incorrect. The brachial artery travels down the midline of the arm, close to the median nerve. It is not immediately endangered by the fracture of the surgical neck. Choice C (Deep brachial, or profunda brachii, artery) is incorrect. The deep brachial artery wraps tightly around the midshaft of the humerus, in the radial (spiral) groove. It, along with the radial nerve with which it travels, would be endangered by a fracture through the radial groove of the humerus. Choice D (Subscapular artery) is incorrect. The subscapular artery is the largest branch of the axillary artery. It descends along the axillary border of the scapula and is not in contact with the humerus. Choice E (Superior ulnar collateral artery) is incorrect. This artery is a branch of the brachial artery. It descends through the arm, moves into company with the ulnar nerve, and takes a close relation to the posterior aspect of the medial epicondyle of the humerus. This vessel would be endangered by a fracture of the medial epicondyle rather than a surgical neck fracture.

32. The given X-ray reveals a fracture of the proximal humerus, indicated by the black arrow. Given the location of the fracture, what artery is most likely damaged in this patient? (A) Posterior circumflex humeral artery (B) Brachial artery (C) Deep brachial (profunda brachii) artery (D) Subscapul arartery (E) Superior ulnar collateral artery

The answer is D. The radial artery divides into the princeps pollicis artery and the deep palmar arterial arch. Thus, stenosis of the radial artery results in a decreased blood flow in the princeps pollicis artery. The superficial palmar arterial arch is formed primarily by the ulnar artery, which passes superficial to the flexor retinaculum. The extensor compart- ment of the forearm receives blood from the posterior interosseous artery, which arises from the common interosseous branch of the ulnar artery. However, the radial and radial recurrent arteries supply the brachioradialis and the extensor carpi radialis longus and brevis.

33. A patient complains of having pain with repeated movements of his thumb (claudication). His physician performs the Allen test and finds an insufficiency of the radial artery. Which of the following conditions would be a result of the radial artery stenosis? (A) A marked decrease in the blood flow in the superficial palmar arterial arch (B) Decreased pulsation in the artery passing superficial to the flexor retinaculum (C) Ischemia of the entire extensor muscles of the forearm (D) A marked decrease in the blood flow in the princeps pollicis artery (E) A low blood pressure in the anterior interosseous artery

The answer is E: Fracture of the fifth metacarpal bone. Fracture of the fifth metacarpal bone is called a "boxer's fracture" because this injury is often seen after an individual improperly punches a solid object with a clenched fist. The impact on the head of the fifth metacarpal causes the distal shaft of this bone to fracture. Skilled pugilists are trained to direct the impact of the clenched fist on the heads of the first and second metacarpals to avoid this type of injury. Choice A (Dislocation of the fifth metacarpophalangeal joint) is incorrect. This type of injury is rare, and a dislocation of the fifth metacarpophalangeal joint would have been detected by the physician on examination. Choice B (Fracture of the triquetral bone) is incorrect. This carpal bone is rarely fractured in this type of impact. Choice C (Fracture of the proximal phalanx of the ring finger) is incorrect. The impact of the clenched fist would have been localized to the head of the fifth metacarpal. Damage to the proximal phalanx of the ring finger would have occurred only if the fist was not clenched during impact. Choice D (Fracture of the proximal phalanx of the little finger) is incorrect. The impact of the clenched fist would have been localized to the head of the fifth metacarpal. Damage to the proximal phalanx of the little finger would have occurred only if the fist was not clenched during impact.

34. A 17-year-old man has pain and moderate swelling over the dorsomedial aspect and in the hypothenar area of his right hand after punching a locker over a dispute with his girlfriend. What is the most likely finding on an X-ray of his hand? (A) Dislocation of the fifth metacarpophalangeal joint (B) Fracture of the triquetral bone (C) Fracture of the proximal phalanx of the ring finger (D) Fracture of the proximal phalanx of the little finger (E) Fracture of the fifth metacarpal bone

The answer is A. The dorsal scapular artery arises directly from the third part of the sub- clavian artery and replaces the deep (descending) branch of the transverse cervical artery. The suprascapular and transverse cervical arteries are branches of the thyrocervical trunk of the subclavian artery. The thoracoacromial artery is a short trunk from the first or second part of the axillary artery and has pectoral, clavicular, acromial, and deltoid branches.

34. A patient bleeding from the shoulder secondary to a knife wound is in fair condition because there is vascular anastomosis around the shoulder. Which of the following arteries is most likely a direct branch of the subclavian artery that is involved in the anastomosis? (A) Dorsal scapular artery (B) Thoracoacromial artery (C) Circumflex scapular artery (D) Transverse cervical artery (E) Suprascapular artery

The answer is B: Supraspinatus tendonitis. This patient is experiencing shoulder impingement syndrome, in which impingement of the supraspinatus tendon leads to supraspinatus tendonitis. Supraspinatus tendonitis is a result of stressful repetitive overhead motions, such as lifting heavy objects, which this patient noted in his summer job. This type of tendonitis presents with a painful arc of motion (between 80 and 150 degrees in this patient) during abduction of the upper limb. Choice A (Infraspinatus tendonitis) is incorrect. Infraspinatus tendonitis would lead to pain during external rotation at the glenohumeral joint against resistance, which is not noted in this patient. Choice C (Acromioclavicular (AC) joint arthritis) is incorrect. AC joint arthritis would result following repeated movements that wear away the cartilage surfaces in the AC joint. It can also be seen after repeated shoulder separations. However, due to the age of this patient and the fact that he was only participating in construction during the summer, this diagnosis is unlikely. Choice D (Degenerative arthritis of the shoulder) is incorrect. Given the young age of this patient, degenerative arthritis of the shoulder is extremely unlikely. Choice E (Broken clavicle) is incorrect. The patient is reporting pain only during abduction of the upper limb. A broken clavicle presents with constant pain and results from trauma; however, trauma was not reported in this patient.

35. A right-handed 21-year-old college student visits his physician because of pain in his right shoulder that developed after starting a summer job on a construction crew 2 weeks ago. He explains that on his job site he regularly lifts heavy construction materials over his head. During physical examination, the patient experiences sharp pain in the range of 80 to 150 degrees of abduction at the glenohumeral joint. What is the most likely diagnosis? (A) Infraspinatus tendonitis (B) Supraspinatus tendonitis (C) Acromioclavicular (AC) joint arthritis (D) Degenerative arthritis of the shoulder (E) Broken clavicle

The answer is A. The musculocutaneous nerve contains sympathetic postganglionic fibers that supply sweat glands and blood vessels on the lateral side of the forearm as the lateral antebrachial cutaneous nerve. The musculocutaneous nerve does not supply the exten- sors of the forearm and the brachioradialis. This nerve also supplies tactile sensation on the lateral side of the forearm but not the arm and supplies blood vessels on the lateral side of the forearm but not the hand.

36. A patient with a stab wound receives a laceration of the musculocutaneous nerve. Which of the following conditions is most likely to have occurred? (A) Lack of sweating on the lateral side of the forearm (B) Inability to extend the forearm (C) Paralysis of brachioradialis muscle (D) Loss of tactile sensation on the arm (E) Constriction of blood vessels on the hand

The answer is E. The pectoralis major is innervated by the lateral and medial pectoral nerves originating from the lateral and medial cords of the brachial plexus, respectively. The subscapularis, teres major, latissimus dorsi, and teres minor muscles are innervated by nerves originating from the posterior cord of the brachial plexus.

37. A 20-year-old man fell from the parallel bar during the Olympic trial. A neurologic examination reveals that he has a lesion of the lateral cord of the brachial plexus. Which of the following muscles is most likely weakened by this injury? (A) Subscapularis (B) Teres major (C) Latissimus dorsi (D) Teres minor (E) Pectoralis major

The answer is E: Flexor digitorum superficialis (FDS). The FDS muscle flexes the proximal interphalangeal joints in digits 2 to 5. In this patient, this muscle is being tested in the third (middle) finger. By holding the other fingers in extension, the physician is eliminating the influences of the FDP muscles, which flex the distal interphalangeal joints in digits 2 to 5. Choice A (Flexor digitorum profundus [FDP]) is incorrect. Flexion of the distal interphalangeal joint in digits 2 to 5 is produced by the FDP; however, the physician is testing flexion of the proximal interphalangeal joint in the middle finger, which is controlled by the FDS muscle. Choice B (Extensor digitorum) is incorrect. The extensor digitorum extends fingers #2 to #5. This muscle can easily be eliminated because the physician is testing a muscle on the flexor side of the hand. Choice C (Second lumbrical) is incorrect. The second lumbrical muscle extends the interphalangeal joints of the middle (third) finger and flexes the metacarpophalangeal joint of the same finger. The third lumbrical is an intrinsic hand muscle that arises off the tendon of the FDP and inserts into the extensor expansion of the middle finger. This muscle does not control flexion of the proximal interphalangeal joint. Choice D (Dorsal interosseous) is incorrect. The primary action of the dorsal interosseous muscles is to abduct the fingers. However, because these muscles insert into the extensor expansion, they also extend the interphalangeal joints of the middle (third) finger and flex the metacarpophalangeal joint of the same finger. Although the dorsal interossei are not involved with flexion of the distal interphalangeal joint, it is important to remember the middle (third) finger forms the axis of movement of the hand. Therefore, this finger has two dorsal interossei attached to it to enable abduction away from the midline in either direction.

37. As part of a physical examination to evaluate intrinsic hand muscle function, a physician holds three fingers in the extended position, and instructs the patient to flex the proximal interphalangeal joint of the free finger, as shown. Which of the following muscles is the doctor specifically testing? (A) Flexor digitorum profundus (FDP) (B) Extensor digitorum (C) Second lumbrical (D) Dorsal interosseous (E) Flexor digitorum superficialis (FDS)

The answer is A: Adduction of the arm. In a traditional (Halsted) radical mastectomy, both the pectoralis major and minor muscles are removed with the breast and associated axillary tissue. The primary actions of the pectoralis major are adduction, flexion, and medial rotation of the arm. These actions are weakened but not lost postoperatively. Other neighboring muscles (e.g., subscapularis, latissimus dorsi, biceps brachii, anterior deltoid) also perform these actions and can compensate somewhat for loss of the pectoralis major. Choice B (Abduction of the arm) is incorrect. Abduction of the arm is performed by the supraspinatus, deltoid, and serratus anterior muscles. Removing the pectoralis major would not affect abduction of the arm. In fact, abduction, extension, and lateral rotation of the arm are the antagonistic actions to the pectoralis major. Choice C (Extension of the arm) is incorrect. The pectoralis major helps with flexion of the arm, not extension (the antagonistic action). The latissimus dorsi and posterior part of the deltoid help with extension of the arm. Choice D (Lateral rotation of the arm) is incorrect. The pectoralis major helps with medial rotation of the arm, not lateral rotation of the arm (the antagonistic action). The infraspinatus, teres minor, and posterior part of the deltoid help with lateral rotation of the arm. Choice E (Depression of the arm) is incorrect. The pectoralis major muscle acts to adduct, flex, and medial rotate the arm. Therefore, a radical mastectomy would affect these actions.

39. The traditional radical mastectomy includes removal of the pectoralis major muscle. Which of the following movements is most affected postoperatively by this surgical procedure? (A) Adduction of the arm (B) Abduction of the arm (C) Extension of the arm (D) Lateral rotation of the arm (E) Depression of the arm

The answer is A: Supraspinatus. The greater tubercle of the humerus is the insertion site of three (of the four) rotator cuff muscles: supraspinatus, infraspinatus, and teres minor. Avulsion of this structure could result in detachment of any of these rotator cuff muscles, depending upon the size and scope of the fracture. However, the wrestler is unable to initiate abduction of the upper limb, which implies damage to the supraspinatus muscle. Choice B (Long head of biceps brachii) is incorrect. This muscle originates from the supraglenoid tubercle of the scapula and passes between the greater and lesser tubercles of the humerus, in the intertubercular (bicipital) groove. Detachment of the tendon of this muscle causes the biceps brachii to bulge in the anterior arm. Avulsion of the biceps brachii muscle is not related to the greater tubercle of the humerus. Choice C (Long head of triceps) is incorrect. The long head of the triceps brachii muscle originates from the infraglenoid tubercle of the scapula and inserts on the olecranon process of the ulna. It would not be involved in avulsion of the greater tubercle of the humerus. Choice D (Subscapularis) is incorrect. The fourth rotator cuff muscle, subscapularis, inserts onto the lesser tubercle of the humerus, so it would not be directly involved with this avulsion injury. Choice E (Infraspinatus) is incorrect. The infraspinatus muscle does insert onto the middle aspect of the greater tubercle of the humerus; however, damage to this muscle would result in weakness in external rotation at the shoulder joint, not the problems with abduction seen in this patient.

4. As a result of chronic stress associated with an intense high school weight-lifting program, a 15-year-old boy suffers an avulsion fracture of the greater tubercle of the humerus. In the ER, he displays difficulty initiating abduction of the upper limb. Which of the following muscles was involved in this fracture? (A) Supraspinatus (B) Long head of biceps brachii (C) Long head of triceps (D) Subscapularis (E) Infraspinatus

The answer is D: Thoracodorsal nerve. Successful transposition of muscle flaps in reconstructive surgery hinges largely on maintaining the neurovascular pedicles that supply the muscle. The latissimus dorsi muscle is innervated by the thoracodorsal (middle subscapular) nerve, a branch of the posterior cord of the brachial plexus. Lesion of this nerve during dissection of the chest wall will cause loss of the muscle flap. Choice A (Long thoracic nerve) is incorrect. The long thoracic nerve supplies the serratus anterior muscle. Damage to this nerve would cause a "winged scapula." Choice B (Intercostobrachial nerve) is incorrect. The intercostobrachial nerve is actually the lateral cutaneous branch of the second thoracic nerve and is entirely sensory in innervation. This nerve arises from the chest wall underneath the second rib and travels distally to give sensory innervation to the skin of the medial arm. Choice C (Medial pectoral nerve) is incorrect. The medial pectoral nerve innervates the pectoralis minor and major muscles. It would have been cut during the removal of these muscles during the radical mastectomy. Choice E (Axillary nerve) is incorrect. The axillary nerve traverses the quadrangular space (accompanying the posterior circumflex humeral artery) to supply the teres minor and deltoid muscles as well as give cutaneous innervation to the upper lateral aspect of the arm.

41. Following a radical mastectomy procedure, a surgeon plans to conduct a breast reconstruction utilizing a latissimus dorsi muscle flap. What nerve will the surgeon need to keep intact during the surgical dissection of the chest wall to prevent atrophy of the muscle flap? (A) Long thoracic nerve (B) Intercostobrachial nerve (C) Medial pectoral nerve (D) Thoracodorsal nerve (E) Axillary nerve

The answer is B: Suspensory Ligaments. Fibrous lobar septa are connective tissue partitions that separate the glandular lobes of the breast and compartmentalize breast tissue. These septa run through the depth of the breast from the dermis of the skin to the underlying pectoral fascia and are most pronounced in the superior aspect of the breast, where they are termed suspensory (Cooper's) ligaments of the breast. Edema and/or tumor growth within the breast can apply traction on the suspensory ligaments, and this tension causes dimpling of the skin that resembles an orange peel. Choice A (Clavipectoral fascia) is incorrect. This sleeve of connective tissue extends from the clavicle to the deep fascia of the axilla and envelopes the subclavius and pectoralis minor muscles. It does not enter the breast. Choice C (Lactiferous ducts) is incorrect. Each of the 15 to 20 glandular lobes of the breast is drained by a lactiferous duct that opens in the lactiferous sinus and then onto the nipple. Edema and/or tumor growth within the breast may apply traction to these ducts. However, the result of such traction is typically inversion of the nipple rather than dimpling of the skin. Choice D (Retromammary space) is incorrect. The retromammary space (or bursa) is a loose connective tissue plane between the breast and pectoral fascia, which contains a small amount of fat and allows the breast some degree of mobility on the pectoral fascia. It does not send connective tissue extensions into the substance of the breast and would not lead to dimpling of the skin due to the presence of an abnormal growth within the breast. Choice E (Pectoralis major muscle) is incorrect. The pectoralis major underlies the breast but does not send fibers into the breast. It does not influence the appearance of the skin of the breast.

42. An 80-year-old woman comes to the physician because of a lump in her right breast. Physical examination shows a 2-cm mass in the right breast with dimpling of the overlying skin and peau d'orange (edema of the breast with the skin assuming the appearance of an orange peel). Examination of a biopsy specimen confirms a diagnosis of carcinoma. Involvement of what structure is the most likely cause of this patient's skin dimpling? (A) Clavipectoral fascia (B) Suspensory ligaments (C) Lactiferous ducts (D) Retromammary space (E) Pectoralis major

The answer is D: Deep branch of ulnar nerve. The deep branch of the ulnar nerve innervates most of the intrinsic muscles of the hand, including the hypothenar muscles, medial two lumbrical muscles, the palmar and dorsal interossei, adductor pollicis, and the deep head of the flexor pollicis brevis. Specifically, the palmar interossei, dorsal interossei, and lumbrical muscles insert into the extensor digital expansion to provide, collectively, flexion of the metacarpophalangeal joints and extension of the interphalangeal joints of the fingers. All of these muscles, with the exception of the first and second lumbrical muscles, are innervated by the deep branch of the ulnar nerve, so this nerve is being tested by placing the hand in the Z-position. If the deep branch of the ulnar nerve were severed, the hand would assume the opposite of the Z-position, which is called "claw hand." Choice A (Deep branch of radial nerve) is incorrect. The radial nerve descends from the arm into the cubital fossa, where it divides into superficial and deep branches. The deep branch of the radial nerve pierces the supinator muscle, winds around the proximal end of the radius within the substance of that muscle, and passes into the deep posterior compartment of the forearm as the posterior interosseous nerve. The nerve may be entrapped within the supinator, resulting in supinator syndrome. Such a condition would affect the deeper, more distal extensor muscles arising in the forearm and some sensory areas in the wrist joints. It is not responsible for the Z position of the hand. Choice B (Superficial branch of radial nerve) is incorrect. The superficial branch of the radial nerve arises from the radial nerve in the cubital fossa. This nerve is purely a sensory nerve, supplying the dorsum of the hand and fingers. Because it does not have a motor component, this nerve would not be responsible for assuming the Z position of the hand. Choice C (Recurrent branch of median nerve) is incorrect. The recurrent (thenar) branch of the median nerve innervates most of the thenar muscles, including the abductor pollicis brevis, opponens pollicis, and superficial head of the flexor pollicis brevis. Cutting this nerve would lead to atrophy of the thenar muscular complex, a condition known as "ape hand." The recurrent branch of the median nerve does not supply muscles outside of the thumb, so it would not be responsible for assuming the hand's Z position. Choice E (Superficial branch of ulnar nerve) is incorrect. The superficial branch of the ulnar nerve arises from the ulnar nerve distal to the flexor retinaculum. This nerve supplies cutaneous branches to the anterior surface of the medial one and a half fingers. The palmaris brevis is the only muscle supplied by this nerve, and this muscle tightens the skin of the medial surface of the palm. Therefore, this nerve would not be responsible for the Z position of the hand.

43. As part of a physical examination to evaluate intrinsic hand muscle function, a physician asks the patient to assume the Z-position (seen in photo) with his hand, which involves flexion of the metacarpophalangeal joints and extension of the interphalangeal joints of the fingers. Which of the following nerves is being tested in assuming this position? (A) Deep branch of radial nerve (B) Superficial branch of radial nerve (C) Recurrent branch of median nerve (D) Deep branch of ulnar nerve (E) Superficial branch of ulnar nerve

The answer is C: Recurrent branch of median nerve. The recurrent (thenar) branch of the median nerve lies subcutaneously in the thenar eminence and can be damaged by lacerations in this area. This nerve innervates most of the thenar muscles, including the Opponens pollicis, Abductor pollicis brevis, and superficial head of the Flexor pollicis brevis (mnemonic = "OAF"). Opposition of the thumb would be lost by cutting the recurrent branch of the median nerve. Though the abductor pollicis brevis is denervated, the abductor pollicis longus, innervated by the deep branch of the radial nerve, is still intact. Also, the flexor pollicis longus, innervated by the anterior interosseous nerve, would still allow flexion at the interphalangeal joint of the thumb. Cutting the recurrent branch of the median nerve would lead to atrophy of the thenar muscular complex, a condition known as "ape hand." Choice A (Deep branch of the radial nerve) is incorrect. The deep branch of the radial nerve is also called the posterior interosseous nerve as it exits from the supinator muscle. It innervates the abductor pollicis longus and extensor pollicis longus and brevis muscles, which form the boundaries of the anatomical snuffbox. Cutting this nerve would cause loss of extension of the thumb and weakness in abduction. However, the puncture was on the thenar eminence, and the radial nerve does not travel there. Choice B (Superficial branch of the radial nerve) is incorrect. The superficial branch of the radial nerve arises from the radial nerve in the cubital fossa. This nerve is purely cutaneous, supplying sensation to the dorsum of the hand and fingers. Because it does not have a motor component, this nerve would not be responsible for the deficit in thumb function. Choice D (Deep branch of ulnar nerve) is incorrect. The deep branch of the ulnar nerve innervates most of the intrinsic muscles of the hand, including the hypothenar muscles, medial two lumbrical muscles, the palmar and dorsal interossei, adductor pollicis, and the deep head of the flexor pollicis brevis. With a small motor supply to the thenar compartment (specifically the deep head of the flexor pollicis brevis), this nerve could not cause a substantial loss of function in the thenar eminence. Choice E (Superficial branch of the ulnar nerve) is incorrect. The superficial branch of the ulnar nerve arises from the ulnar nerve distal to the flexor retinaculum. This nerve supplies cutaneous branches to the anterior surface of the medial one and a half fingers. The palmaris brevis is the only muscle supplied by this nerve, and this muscle tightens the skin of the medial surface of the palm. Therefore, this nerve would not be responsible for substantial loss of motor function in the thumb.

44. A 10-year-old boy was running across a parking lot when he tripped and received lacerations on the base of his thumb from a broken glass bottle. On examination, his thumb was unable to oppose to his fingers, and the thumb also showed weakness when abducting and flexing. No sensory deficits were reported. What nerve was most likely severed? (A) Deep branch of radial nerve (B) Superficial branch of radial nerve (C) Recurrent branch of median nerve (D) Deep branch of ulnar nerve (E) Superficial branch of ulnar nerve

The answer is B: Anterior axillary lymph nodes. The primary lymphatic drainage route for the lateral half of the breast (including the axillary tail) plus part of the medial half of the breast is first into the anterior (pectoral; Level 1) axillary lymph nodes. These lymph nodes are located along the lateral border of the pectoralis minor muscle. Further metastasis would spread progressively to the central axillary (Level 2) nodes, then the apical axillary (Level 3) nodes. Choice A (Lateral axillary lymph nodes) is incorrect. These nodes, located along the distal segment of the axillary vein, collect the lymphatic drainage from most of the upper limb. They drain to the central axillary (Level 2) nodes. Choice C (Deep cervical lymph nodes) is incorrect. These nodes are located in the neck, along the internal jugular vein, and collect lymph from the head and neck. Choice D (Parasternal lymph nodes) is incorrect. The parasternal (internal thoracic) nodes lie along the internal thoracic vessels within the chest. Most of the medial half of the breast sends its lymphatic drainage primarily to these nodes. Choice E (Contralateral breast lymph nodes) is incorrect. Small areas of the medial aspect of the breast may send an accessory lymphatic drainage across the midline to the contralateral nodes of the opposite breast. Due to the site of the tumor, this lymphatic drainage pattern is unlikely.

45. A 48-year-old woman is diagnosed with a malignant tumor in the superolateral quadrant of the right breast, including the axillary tail. If it metastasizes, this cancer will most likely spread first to which of the following locations? (A) Lateral axillary lymph nodes (B) Anterior axillary lymph nodes (C) Deep cervical lymph nodes (D) Parasternal lymph nodes (E) Contralateral breast lymph nodes

The answer is D: Weakness extending the wrist and fingers. The radial nerve is particularly vulnerable during midshaft humeral fractures because it is located directly against the bone in this region. The radial nerve innervates the extensor muscles of the arm and forearm and also carries sensory innervation from the posterior aspect of the arm, forearm, and hand and the lateral aspect of the arm (but not the forearm). Depending upon the exact placement of the fracture, the nerve fibers that innervate the triceps brachii may have already left the radial nerve. However, the fibers innervating the extensors of the wrist and fingers would still be bundled in the radial nerve and would be vulnerable to damage during a fracture of this type. Choice A (Numbness on the lateral [radial] aspect of the forearm) is incorrect. The radial nerve innervates the skin on the posterior aspect of the arm and forearm and the lateral aspect of the lower arm. The lateral (radial) aspect of the forearm is innervated primarily by the lateral cutaneous nerve of the forearm, the terminal branch of the musculocutaneous nerve. Choice B (Numbness of the medial aspect of the upper arm) is incorrect. The skin over the medial aspect of the upper arm is innervated by the medial cutaneous nerve of the arm (medial brachial cutaneous nerve) that originates from the medial cord of the brachial plexus. This nerve would be protected from the humeral fracture due to its location. Choice C (Numbness over the superolateral aspect of the upper arm) is incorrect. The cutaneous innervation of the superolateral aspect of the upper arm is derived from the axillary nerve that shares a common origin with the radial nerve from the posterior cord of the brachial plexus. Though both nerves arise from the posterior cord, the axillary and radial nerves separate proximal to the site of the fracture, thus leaving the axillary nerve undamaged by the fracture. Choice E (Weakness in grip strength) is incorrect. While the pain associated with the fracture would affect the integrity of all muscle activity in the limb, all of the muscles used for gripping (flexing the wrist and digits) are innervated by the median and ulnar nerves, which do not have close relations to the fracture site.

46. A 48-year-old woman falls on an icy sidewalk and lands on her right elbow. She suffers a midshaft humeral fracture, as seen on the given X-ray. The attending physician wants to assess whether the nerve residing in the spiral groove of the humerus is damaged. What sign or symptom would confirm damage to this nerve? (A) Numbness on the lateral (radial) aspect of the forearm (B) Numbness of the medial aspect of the upper arm (C) Numbness over the superolateral aspect of the upper arm (D) Weakness extending the wrist and fingers (E) Weakness in grip strength

The answer is A: Pectoralis minor. The pectoralis minor muscle forms a portion of the anterior axillary wall and is used as a marker to organize the contained groups of axillary lymph nodes. In a clinical context, the axillary nodes are organized into three major groups (Level 1, 2, and 3) that denote the progressive lymph drainage of the breast. Level 1 (anterior; pectoral) nodes are located along the lateral border of the pectoralis minor. Level 2 (central; deep) nodes are situated in the center of the axilla, deep (posterior) to the pectoralis minor. Level 3 (apical; medial) nodes are positioned in the apex of the axilla, superior to the upper medial border of the pectoralis minor. Detection of cancer cells in each increasing number level of nodes indicates progressively greater metastasis of the disease out of the breast. Choice B (Pectoralis major) is incorrect. The pectoralis major and minor form the anterior axillary wall. However, the pectoralis major muscle is not used to classify the positions of the axillary lymph nodes or levels of metastasis. Choice C (Latissimus dorsi) is incorrect. The latissimus dorsi muscle helps to form the posterior wall of the axilla along with the subscapularis and teres major muscles. Due to its posterior location, it is not an important landmark for classification of axillary lymph nodes in breast cancer. Choice D (Serratus anterior) is incorrect. This muscle contributes to the formation of the medial wall of the axilla, along with the thoracic cage. This muscle is not an important landmark for classification of axillary lymph nodes in breast cancer. Choice E (Subscapularis) is incorrect. The subscapularis muscle helps to form the posterior wall of the axilla along with the latissimus dorsi and teres major muscles. Due to its posterior location, this muscle is not an important landmark for classification of axillary lymph nodes in breast cancer.

47. Organization of the axillary lymph nodes into Levels I, II, and III for breast cancer treatment is based on the location of the nodes relative to which of the following muscles? (A) Pectoralis minor (B) Pectoralis major (C) Latissimus dorsi (D) Serratus anterior (E) Subscapularis

The answer is E: Scaphoid. The proximal segment of the scaphoid bone has a poor supply of blood because the palmar carpal branch of the radial artery enters the distal part of the scaphoid and then supplies blood proximally. This small artery is often severed during fractures of the scaphoid bone leading to avascular necrosis, or death of the bone due to poor blood supply. Remember, the scaphoid is the most commonly fractured carpal bone, but a fracture within this bone is often not seen on initial radiographs. Radiographs taken several weeks later will show the fracture due to bone resorption at the fracture site. Choice A (Distal radius) is incorrect. The distal radius is often fractured in older individuals who fall on their outstretched hand. This fracture of the distal radius is called a Colles fracture ("dinner fork deformity") due to its appearance on a lateral radiograph. However, the distal radius has an adequate blood supply and is not prone to avascular necrosis. Choice B (Midshaft ulna) is incorrect. A fracture of the midshaft ulna is not prone to avascular necrosis due to its adequate blood supply. Choice C (Fifth metacarpal) is incorrect. The fifth metacarpal is often fractured when an individual improperly punches a solid object with a clenched fist, as in a "boxer's fracture." This bone is not prone to avascular necrosis due to its adequate blood supply. Choice D (Lunate) is incorrect. The lunate is the most commonly dislocated carpal bone, which leads to severe carpal instabilities. This dislocation often occurs in association with a trans-scaphoid fracture. If this bone were fractured in this work-related accident, it would not be prone to avascular necrosis due to its adequate blood supply.

48. A 37-year-old factory worker fractures multiple bones distal to the elbow when his hand and forearm are crushed by equipment dropped by a faulty hydraulic lift. Which of the following bones, if fractured, would most likely develop avascular necrosis? (A) Distal radius (B) Midshaft ulna (C) Fifth metacarpal (D) Lunate (E) Scaphoid

The answer is C: Dorsal interosseous muscles. The primary action of the dorsal interossei is to abduct the fingers, which is being tested in this photo. It is also important to remember the dorsal interossei insert into the extensor digital expansion, so these muscles work with the palmar interossei and the lumbrical muscles to extend the interphalangeal joints and flex the metacarpophalangeal joints. The nerve being tested in this patient, via abduction of the fingers, is the deep branch of the ulnar nerve. A mnemonic is "DAB", which stands for Dorsal interossei ABduct the fingers. Choice A (Lumbrical Muscles) is incorrect. The lumbrical muscles arise off the flexor digitorum profundus tendons and insert into the lateral (radial) aspect of the extensor digital expansion. Due to their course and insertion, the lumbrical muscles extend the interphalangeal joints and flex the metacarpophalangeal joints. These intrinsic hand muscles are not being tested when a patient abducts the fingers against resistance. Choice B (Palmar interosseous muscles) is incorrect. The primary action of the three palmar interossei is to adduct the index finger, ring finger, and little finger toward the axis of the hand, which is the middle finger. The deep branch of the ulnar nerve supplies the palmar interossei. A mnemonic is "PAD", which stands for Palmar interossei ADduct the fingers. It is also important to remember the palmar interossei insert into the extensor digital expansion, so these muscles work in concert with the dorsal interossei and the lumbrical muscles to extend the interphalangeal joints and flex the metacarpophalangeal joints of the fingers. Choice D (Flexor digitorum superficialis) is incorrect. The flexor digitorum superficialis muscle flexes the proximal interphalangeal joint in digits 2 to 5, and influences the distal interphalangeal joint by binding the tendons of the flexor digitorum profundus muscle. Flexion of the proximal interphalangeal joints of the fingers is not being tested in this patient. Choice E (Flexor digitorum profundus) is incorrect. The flexor digitorum profundus muscle is responsible for flexion of the distal interphalangeal joint in digits 2 to 5; however, this action is not being tested in this photo.

49. As part of a physical examination to evaluate muscle function in the hand, a physician holds the four fingers (digits 2 through 5) and asks the patient to spread their fingers, as shown below. What muscle(s) is/are the doctor testing? (A) Lumbrical muscles (B) Palmar interosseous muscles (C) Dorsal interosseous muscles (D) Flexor digitorum superficialis (E) Flexor digitorum profundus

The answer is E: Inability to adduct the thumb. The location of the cut, superficial to the flexor retinaculum, indicates injury to the ulnar nerve. This nerve controls most of the intrinsic muscles of the hand, including the adductor pollicis. Because this muscle is the sole adductor of the thumb, the result will be inability to adduct that digit. Choice A (Weakness in pronation) is incorrect. Pronation is controlled by branches of the median nerve to the pronator teres and pronator quadratus muscles. These are located proximal to the site of the injury. Choice B (Inability to abduct the thumb) is incorrect. Abduction of the thumb is governed by the radial nerve (to abductor pollicis longus) and the recurrent branch of the median nerve (to abductor pollicis brevis) at sites removed from the injury. Choice C (Weakness in flexion of the thumb) is incorrect. Flexion of the thumb is controlled by branches of the median nerve at locations proximal (to flexor pollicis longus) and distal (to flexor pollicis brevis) to the cut flexor retinaculum (transverse carpal ligament). However, the reported laceration was superficial to this ligament and proximal to the pisiform bone, so the median nerve would not have been damaged. Choice D (Weakness in opposition of the thumb) is incorrect. Opposition of the thumb is controlled by the recurrent branch of the median nerve (to opponens pollicis), distal to the injury site.

5. During an attempted suicide, a depressed young woman slashes her wrist with a straight razor. She cuts just proximal to the pisiform bone to the depth of the superficial aspect of the flexor retinaculum before passing out at the sight of her own blood. As a result of this wound, she may suffer a neuromuscular deficit that results in which of the following? (A) Weakness in pronation (B) Inability to abduct the thumb (C) Weakness in flexion of the thumb (D) Weakness in opposition of the thumb (E) Inability to adduct the thumb

The answer is E: Anular ligament of radius. The anular ligament of the radius encircles and holds the head of the radius in the radial notch of the ulna (see diagram on next page). This ligament enables pronation and supination of the forearm. However, the head of the radius can be pulled distally out of this anular ligament resulting in a subluxation or dislocation of the radial head, which is frequently called "nursemaid's elbow." This injury is often seen in children, particularly girls, between the ages of 1 to 3 years old. It occurs when an extended arm is pulled, commonly during a fall, and the individual holding the hand does not let go, as reported in this case. Subluxation and dislocation of the radial head are also seen when the child is swinging while being held by the hands. Choice A (Interosseous membrane of the forearm) is incorrect. The interosseous membrane of the forearm connects the interosseous borders of the radius and ulna. This membrane would not be injured in this type of the injury, especially because the patient complains of pain in the elbow region. Choice B (Quadrate ligament) is incorrect. The quadrate ligament passes from the distal margin of the radial notch of the ulna to the neck of the radius. Damage to this ligament is rare in this type of injury when the extended arm is pulled. Choice C (Radial collateral ligament of elbow) is incorrect. The radial collateral ligament of the elbow extends from the lateral epicondyle of the humerus to the anular ligament of the radius. This ligament would be injured in forced adduction of the elbow joint, which was not seen in this patient. Choice D (Ulnar collateral ligament of elbow) is incorrect. The ulnar collateral ligament of the elbow extends from the medial epicondyle of the humerus to the coronoid process and olecranon of the ulna. This ligament would be injured in forced abduction of the elbow joint, which was not seen in this patient. This ligament is also frequently injured in athletes who use an overhead throwing motion. When the athlete releases the ball during their throwing motion, the ulna is pulled from the humerus, which can damage this ligament. American baseball pitchers often need surgery to repair the ulnar collateral ligament of the elbow, which is commonly termed "Tommy John surgery."

51. A 3-year-old girl is brought to the emergency room holding her right arm with the elbow flexed and the forearm pronated. She refuses to move her arm and complains her elbow "hurts a lot." Her mother reports they were holding hands and running in the park when the child tripped. The mother pulled on the child's hand to prevent her from hitting the ground. Given the nature of this injury and the age of the patient, what structure is most likely damaged? (A) Interosseous membrane of forearm (B) Quadrate ligament (C) Radial collateral ligament of elbow (D) Ulnar collateral ligament of elbow (E) Anular ligament of radius

The answer is A. Injury to the radial nerve results in loss of wrist extension, leading to wrist drop. The median nerve innervates the pronator teres, pronator quadratus, and opponens pollicis muscles and the skin over the ventral aspect of the thumb. The ulnar nerve innervates the dorsal interosseous muscles, which act to abduct the fingers.

7. A 16 year-old boy fell from a motorcycle, and his radial nerve was severely damaged because of a fracture of the midshaft of the humerus. Which of the following conditions would most likely result from this accident? (A) Loss of wrist extension leading to wrist drop (B) Weakness in pronating the forearm (C) Sensory loss over the ventral aspect of the base of the thumb (D) Inability to oppose the thumb (E) Inability to abduct the fingers

The answer is B: Torn coracoclavicular ligament. "Shoulder separation" describes a dislocation of the acromioclavicular joint. In its most severe form (grade 3), the condition includes a tearing of both the intrinsic acromioclavicular ligament and the extrinsic coracoclavicular ligament. As a result, the scapula separates from the clavicle and falls away due to the weight of the upper limb. Thus, the distal end of the clavicle is prominent. Choice A (Dislocated head of the humerus) is incorrect. Dislocations of the GH joint easily occur inferiorly due to its lack of muscular and ligamentous support. Thus, damage to the axillary nerve often occurs following inferior displacment of the head of humerus from the GH joint. However, the acromioclavicular joint, which is more proximal, was injured in this patient. Dislocations of the glenohumeral joint in other directions are more difficult (but not impossible) because of the support of the rotator cuff muscles (anteriorly and posteriorly) and the coracoacromial arch (superiorly). Choice C (Fractured clavicle) is incorrect. Radiological imaging would have detected a fractured clavicle, but these tests confirmed a shoulder separation and not a fractured clavicle. Choice D (Dislocated sternal end of the clavicle) is incorrect. Due to its intrinsic strength, dislocation of the sternoclavicular (SC) joint is rare. Most dislocations of the SC joint occur in persons less than 25 years of age following a fracture of the epiphysial plate of the clavicle. The epiphysis at the proximal end of the clavicle does not close until approximately age 25. Though this patient was under the age of 25, his injury was localized to the acromioclavicular joint. Choice E (Torn anterior glenohumeral [GH] ligament) is incorrect. Three GH ligaments reinforce the anterior part of the joint capsule; however, the GH joint was not involved in this patient.

7. A 17-year-old boy comes to the emergency room after a hard fall onto the lateral aspect of his left shoulder during a high school basketball game. He complains of generalized pain during shoulder motion. On physical examination, the distal end of the clavicle is prominent and distinctly palpable. Radiological findings confirm the diagnosis of a severe (grade 3) shoulder separation. Which of the following features is a component of this condition? (A) Dislocated head of the humerus (B) Torn coracoclavicular ligament (C) Fractured clavicle (D) Dislocated sternal end of the clavicle (E) Torn anterior glenohumeral (GH) ligament

The answer is E. The extensor digitorum extends both the metacarpophalangeal and interphalangeal joints.

73. Atrophy of this structure impairs extension of both the metacarpophalangeal and interphalangeal joints.

The answer is D. The flexor digitorum superficialis muscle flexes the proximal inter- phalangeal joints. The flexor digitorum profundus muscle flexes the DIP joints. The pal- mar and dorsal interossei and lumbricals can flex metacarpophalangeal joints and extend the interphalangeal joints. The palmar interossei adduct the fingers, and the dorsal inter- ossei abduct the fingers.

8. A patient comes in complaining that she cannot flex her proximal interphalangeal joints. Which of the following muscles appear(s) to be paralyzed on further examination of her finger? (A) Palmar interossei (B) Dorsal interossei (C) Flexor digitorum profundus (D) Flexor digitorum superficialis (E) Lumbricals

The answer is C: Long thoracic nerve. The photo demonstrates a case of "winged scapula," indicative of a lesion of the long thoracic nerve and subsequent paralysis of the serratus anterior muscle. The nerve runs down the lateral thoracic wall, on the superficial aspect of the serratus anterior, where it is unusually exposed (for a motor nerve) and vulnerable to injury, especially when the limb is elevated. Lesion of the nerve denervates the serratus anterior. This results in the medial border and inferior angle of the scapula pulling away from the posterior chest wall, giving the scapula a wing-like appearance when the affected limb is protracted. Additionally, the affected arm cannot be abducted above the horizontal plane because the serratus anterior is not available to superiorly rotate the glenoid cavity of the scapula to allow full abduction. Choice A (Axillary nerve) is incorrect. This nerve passes deeply through the axilla, around the surgical neck of the humerus, to supply the teres minor and deltoid muscles. Lesion here would result in significant weakness in abduction of the arm and wasting of the rounded contour of the shoulder. Choice B (Thoracodorsal nerve) is incorrect. This nerve runs inferior through the axilla to supply the latissimus dorsi muscle. Loss of the nerve would result in weakness in extension and medial rotation of the arm, plus wasting of the posterior axillary fold. Choice D (Dorsal scapular nerve) is incorrect. This nerve courses into the upper, medial part of the back and the lower neck to supply the levator scapulae and rhomboid muscles. Paralysis of these muscles would result in weakness in elevation and retraction of the scapula and perhaps wasting of the contour of the back under the trapezius muscle. Choice E (Suprascapular nerve) is incorrect. The suprascapular nerve runs through the suprascapular notch and into the supraspinous and infraspinous fossae to supply the supraspinatus and infraspinatus muscles. Lesion of this nerve would result in weakness in the rotator cuff affecting the initiation of abduction (supraspinatus) and external rotation (infraspinatus) of the shoulder and wasting of the muscular contour of the posterior aspect of the scapula.

8. An 18-year-old boy is cut severely on the lateral wall of his right chest during a knife fight. Following healing, his scapula moves away from the thoracic wall when he leans on his right hand, giving the appearance indicated in the given photo. Which of the following nerves is likely damaged? (A) Axillary nerve (B) Thoracodorsal nerve (C) Long thoracic nerve (D) Dorsal scapular nerve (E) Suprascapular nerve

The answer is C. The latissimus dorsi adducts the arm, and the supraspinatus muscle abducts the arm. The infraspinatus and the teres minor rotate the arm laterally. The serra- tus anterior rotates the glenoid cavity of the scapula upward, abducts the arm, and ele- vates it above a horizontal position.

9. A 21-year-old woman walks in with a shoulder and arm injury after falling during horseback riding. Examination indicates that she cannot adduct her arm because of paralysis of which of the following muscles? (A) Teres minor (B) Supraspinatus (C) Latissimus dorsi (D) Infraspinatus (E) Serratus anterior

The answer is B: Hamate. The hamate bone is identified by the white arrow in the given X-ray. This bone resides in the medial (ulnar) aspect of the distal row of carpal (wrist) bones. Stress fractures can occur to this bone, particularly within its hook, which appears as a radiodense oval on this radiograph. This type of fracture is frequently seen in golfers due to the positioning of the proximal aspect of the golf club within their grip. A fracture of the hamate results in pain, which is exacerbated by gripping and point tenderness in the skin overlying the bone. The numbness and weakness in the medial aspect of the hand, seen in this patient, are due to impingement of the ulnar nerve. Remember, the eight carpal (wrist) bones form two rows that contain four bones each. The proximal row (from lateral to medial) is composed of the scaphoid, lunate, triquetrum, and pisiform. The distal row (from lateral to medial) is composed of the trapezium, trapezoid, capitate, and hamate. To remember the carpal bones as listed, some students use the mnemonic: "Some Lovers Try Positions That They Can't Handle". In a standard posteroanterior plane film, the medial aspect of the wrist can be deceiving to the untrained eye. Here, the pisiform overlies the triquetrum, and the seemingly single radiopacity might not be distinguished as two separate bones. Instead, the distinctive hook of the hamate bone may be mistaken for the pisiform. Choice A (Capitate) is incorrect. The capitate (L: head) is a head-shaped bone that is the largest of the carpal bones. Its position is noted by its articulation with the third metacarpal distally. Choice C (Lunate) is incorrect. The lunate (L: moon) is a moon-shaped bone between the scaphoid and triquetral bones. It articulates with the radius proximally, and its position makes it the most commonly dislocated carpal bone. Choice D (Pisiform) is incorrect. The pisiform (L: pea) is a pea-shaped sesamoid bone that lies on the palmar aspect of the triquetrum bone. Choice E (Scaphoid) is incorrect. The scaphoid (G: boat) is a boat-shaped bone that is the largest of the carpal bones located in the proximal row. Its articulation with the radius proximally makes it the most commonly fractured carpal bone, especially when a person falls and impacts an abducted, outstretched hand.

9. On his downswing, an amateur golfer strikes the hard earth with his club and feels pain in his right wrist. During a subsequent physical examination, he complains of wrist pain that is exacerbated by gripping, displays point tenderness in his medial wrist, and complains of numbness and weakness in his pinky finger (fifth digit). What carpal bone, identified by the white arrow on the given X-ray, is most likely fractured in this patient? (A) Capitate (B) Hamate (C) Lunate (D) Pisiform (E) Scaphoid

The answer is D. The sternocleidomastoid muscle is attached to the superior border of the medial third of the clavicle, and the medial fragment of a fractured clavicle is displaced upward by the pull of the muscle.

Questions 45-47: A 37-year-old female patient has a fracture of the clavicle. The junction of the middle and lateral thirds of the bone exhibits overriding of the medial and lateral fragments. The arm is rotated medially, but it is not rotated laterally. 46. Which of the following muscles causes upward displacement of the medial fragment? (A) Pectoralis major (B) Deltoid (C) Trapezius (D) Sternocleidomastoid (E) Scalenus anterior

The answer is B. The fractured clavicle may damage the subclavian vein, resulting in a pulmonary embolism; cause thrombosis of the subclavian artery, resulting in embolism of the brachial artery; or damage the lower trunk of the brachial plexus.

Questions 45-47: A 37-year-old female patient has a fracture of the clavicle. The junction of the middle and lateral thirds of the bone exhibits overriding of the medial and lateral fragments. The arm is rotated medially, but it is not rotated laterally. 47. Which of the following conditions is most likely to occur secondary to the fractured clavicle? (A) A fatal hemorrhage from the brachiocephalic vein (B) Thrombosis of the subclavian vein, causing a pulmonary embolism (C) Thrombosis of the subclavian artery, causing an embolism in the ascending aorta (D) Damage to the upper trunk of the brachial plexus (E) Damage to the long thoracic nerve, causing the winged scapula

The answer is E. The lateral fragment of the clavicle is displaced downward by the pull of the deltoid muscle and gravity. The medial fragment is displaced upward by the pull of the sternocleidomastoid muscle. None of the other muscles are involved.

Questions 45-47: A 37-year-old female patient has a fracture of the clavicle. The junction of the middle and lateral thirds of the bone exhibits overriding of the medial and lateral fragments. The arm is rotated medially, but it is not rotated laterally. 45. The lateral portion of the fractured clavicle is displaced downward by which of the following? (A) Deltoid and trapezius muscles (B) Pectoralis major and deltoid muscles (C) Pectoralis minor muscle and gravity (D) Trapezius and pectoralis minor muscles (E) Deltoid muscle and gravity

The answer is E. The fingers are adducted by the palmar interosseous muscles; abduction is performed by the dorsal interosseous muscles. The palmar and dorsal interosseous muscles are innervated by the ulnar nerve. The proximal interphalangeal joints are flexed by the flexor digitorum superficialis, which is innervated by the median nerve. However, the DIP joints of the index and middle fingers are flexed by the flexor digitorum profun- dus, which is innervated by the median nerve (except the medial half of the muscle, which is innervated by the ulnar nerve). The median nerve supplies sensory innervation on the palmar aspect of the middle finger. The abductor pollicis brevis is innervated by the median nerve; the abductor pollicis longus is innervated by the radial nerve.

Questions 48-50: A 21-year-old man injures his right arm in an automobile accident. Radiographic examination reveals a fracture of the medial epicondyle of the humerus. 50. After this injury, the patient is unable to do which of the following? (A) Flex the proximal interphalangeal joint of his ring finger (B) Flex the DIP joint of his index finger (C) Feel sensation on his middle finger (D) Abduct his thumb (E) Adduct his index finger

The answer is B. The axillary nerve runs posteriorly around the surgical neck of the humerus and is vulnerable to injury such as fracture of the surgical neck of the humerus or inferior dislocation of the humerus. The other nerves listed are not in contact with the surgical neck of the humerus.

Questions 51-55: A 10-year-old boy falls off his bike, has difficulty in moving his shoulder, and is brought to an emergency department. His radiogram and angiogram reveal fracture of the surgical neck of his humerus and bleeding from the point of the fracture. 51. Which of the following nerves is most likely injured as a result of this accident? (A) Musculocutaneous (B) Axillary (C) Radial (D) Median (E) Ulnar

The answer is C. The (injured) axillary nerve contains GSE fibers whose cell bodies are located in the anterior horn of the spinal cord, and these GSE fibers supply the deltoid and teres minor muscles. The axillary nerve also contains GSA and GVA fibers, whose cell bodies are located in the dorsal root ganglia, and sympathetic postganglionic fibers, whose cell bodies are located in sympathetic chain ganglia. The lateral horn of the spinal cord between T1 and L2 contains cell bodies of sympathetic preganglionic fibers. The posterior horn of the spinal cord contains cell bodies of interneurons.

Questions 51-55: A 10-year-old boy falls off his bike, has difficulty in moving his shoulder, and is brought to an emergency department. His radiogram and angiogram reveal fracture of the surgical neck of his humerus and bleeding from the point of the fracture. 52. Following this accident, the damaged nerve causes difficulty in abduction, extension, and lateral rotation of his arm. Cell bodies of the injured nerve involved in movement of his arm are located in which of the following structures? (A) Dorsal root ganglion (B) Sympathetic chain ganglion (C) Anterior horn of the spinal cord (D) Lateral horn of the spinal cord (E) Posterior horn of the spinal cord

The answer is D. Axillary nerve contains GSE, GSA, GVA, and sympathetic postganglionic GVE fibers. Cell bodies of GSA and GVA fibers are located in the dorsal root ganglia. Cell bodies of GSE fibers are located in the anterior horn of the spinal cord. Cell bodies of sympathetic postganglionic GVE fibers are located in the sympathetic chain ganglia, but cell bodies of sympathetic preganglionic GVE fibers lie in the lateral horn of the spinal cord.

Questions 51-55: A 10-year-old boy falls off his bike, has difficulty in moving his shoulder, and is brought to an emergency department. His radiogram and angiogram reveal fracture of the surgical neck of his humerus and bleeding from the point of the fracture. 53. The damaged nerve causes numbness of the lateral side of the arm. Cell bodies of the injured nerve fibers involved in sensory loss are located in which of the following structures? (A) Anterior horn of the spinal cord (B) Posterior horn of the spinal cord (C) Lateral horn of the spinal cord (D) Dorsal root ganglia (E) Sympathetic chain ganglia

The answer is C. The posterior humeral circumflex artery accompanies the axillary nerve that passes around the surgical neck of the humerus. None of the other arteries are involved.

Questions 51-55: A 10-year-old boy falls off his bike, has difficulty in moving his shoulder, and is brought to an emergency department. His radiogram and angiogram reveal fracture of the surgical neck of his humerus and bleeding from the point of the fracture. 54. This accident most likely leads to damage of which of the following arteries? (A) Axillary (B) Deep brachial (C) Posterior humeral circumflex (D) Superior ulnar collateral (E) Scapular circumflex

The answer is B. The lateral rotators of the arm include the teres minor, deltoid, and infra- spinatus muscles, but the infraspinatus muscle is innervated by the suprascapular nerve.

Questions 51-55: A 10-year-old boy falls off his bike, has difficulty in moving his shoulder, and is brought to an emergency department. His radiogram and angiogram reveal fracture of the surgical neck of his humerus and bleeding from the point of the fracture. 55. Following this accident, the boy has weakness in rotating his arm laterally. Which of the following muscles are paralyzed? (A) Teres major and teres minor (B) Teres minor and deltoid (C) Infraspinatus and deltoid (D) Supraspinatus and subscapularis (E) Teres minor and infraspinatus

The answer is A. The median cubital vein lies superficial to the bicipital aponeurosis and thus separates it from the brachial artery, which can be punctured during intravenous injections and blood transfusions.

Questions 56 and 57: A 64-year-old man with a history of liver cirrhosis has been examined for hepatitis A, B, and C viruses. In an attempt to obtain a blood sample from the patient's median cubital vein, a registered nurse inadvertently procures arterial blood. 56. The blood most likely comes from which of the following arteries? (A) Brachial (B) Radial (C) Ulnar (D) Common interosseous (E) Superior ulnar collateral

The answer is B. The median nerve is damaged because it lies medial to the brachial artery. The bicipital aponeurosis lies on the brachial artery and the median nerve. The V-shaped cubital fossa contains (from medial to lateral) the median nerve, brachial artery, biceps tendon, and radial nerve. The ulnar nerve runs behind the medial epicondyle; the lateral and medial antebrachial cutaneous nerves are not closely related to the brachial artery.

Questions 56 and 57: A 64-year-old man with a history of liver cirrhosis has been examined for hepatitis A, B, and C viruses. In an attempt to obtain a blood sample from the patient's median cubital vein, a registered nurse inadvertently procures arterial blood. 57. During the procedure, the needle hits a nerve medial to the artery. Which of the following nerves is most likely damaged? (A) Radial (B) Median (C) Ulnar (D) Lateral antebrachial (E) Medial antebrachial

The answer is D. The (damaged) radial nerve contains sympathetic postganglionic nerve fibers whose cell bodies are located in the sympathetic chain ganglion. Sympathetic postganglionic fibers supply sweat glands, blood vessels, and hair follicles (arrector pili muscles). The radial nerve also contains GSE fibers whose cell bodies are located in the anterior horn of the spinal cord, and GSA and GVA fibers whose cell bodies are located in the dorsal root ganglion. The lateral horn of the spinal cord between T1 and L2 contains cell bodies of sympathetic preganglionic nerve fibers.

Questions 58-62: A 17-year-old boy is injured in an automobile accident. He has a fracture of the shaft of the humerus. 59. As a result of this fracture, the patient shows lack of sweating on the back of the arm and forearm. Cell bodies of the damaged nerve fibers involved in sweating are located in which of the following structures? (A) Anterior horn of the spinal cord (B) Posterior horn of the spinal cord (C) Lateral horn of the spinal cord (D) Sympathetic chain ganglion (E) Dorsal root ganglion

The answer is D. The superficial branch of the radial nerve runs distally to the dorsum of the hand to innervate the radial side of the hand, including the area of the anatomic snuff- box and the radial two and one-half digits over the proximal phalanx. The medial aspect of the arm is innervated by the medial brachial cutaneous nerve; the lateral aspect of the forearm is innervated by the lateral antebrachial cutaneous nerve of the musculocutane- ous nerve; the palmar aspect of the second and third digits is innervated by the median nerve; and the medial one and one-half fingers are innervated by the ulnar nerve.

Questions 58-62: A 17-year-old boy is injured in an automobile accident. He has a fracture of the shaft of the humerus. 60. Following this accident, the patient has no cutaneous sensation in which of the following areas? (A) Medial aspect of the arm (B) Lateral aspect of the forearm (C) Palmar aspect of the second and third digits (D) Area of the anatomic snuffbox (E) Medial one and one-half fingers

The answer is C. The radial nerve accompanies the profunda brachii artery in the radial groove on the posterior aspect of the shaft of the humerus. The posterior humeral circum- flex artery accompanies the axillary nerve around the surgical neck of the humerus. Other arteries are not associated with the radial groove of the humerus.

Questions 58-62: A 17-year-old boy is injured in an automobile accident. He has a fracture of the shaft of the humerus. 61. Which of the following arteries may be damaged? (A) Brachial artery (B) Posterior humeral circumflex artery (C) Profunda brachii artery (D) Radial artery (E) Radial recurrent artery

The answer is D. A lesion of the radial nerve causes paralysis of the supinator and brachio- radialis. The biceps brachii muscle is a flexor of the elbow and also a strong supinator; thus, supination is still possible through action of the biceps brachii muscle. Other mus- cles cannot supinate the forearm.

Questions 58-62: A 17-year-old boy is injured in an automobile accident. He has a fracture of the shaft of the humerus. 62. After this accident, supination is still possible through contraction of which of the following muscles? (A) Supinator (B) Pronator teres (C) Brachioradialis (D) Biceps brachii (E) Supraspinatus

The answer is D. The scapular notch transmits the suprascapular nerve below the superior transverse ligament, whereas the suprascapular artery and vein run over the ligament. The suprascapular nerve supplies the supraspinatus and infraspinatus muscles. The axillary nerve innervates the deltoid and teres minor muscles. The subscapular nerves innervate the teres major and subscapularis muscles.

Questions 63 and 64: An 11-year-old boy falls down the stairs. A physician examines a radiograph of the boy's shoulder region (see figure below). 63. If the structure indicated by the letter A is calcified, which of the following muscles is most likely paralyzed? (A) Deltoid (B) Teres major (C) Teres minor (D) Infraspinatus (E) Subscapularis"

The answer is D. Fracture of the surgical neck of the humerus occurs commonly and dam- ages the axillary nerve and the posterior humeral circumflex artery.

Questions 63 and 64: An 11-year-old boy falls down the stairs. A physician examines a radiograph of the boy's shoulder region (see figure below). 64. If the structure indicated by the letter B is fractured, which of the following structures is most likely injured? (A) Musculocutaneous nerve (B) Radial nerve (C) Deep brachial artery (D) Posterior humeral circumflex artery (E) Scapular circumflex artery

The answer is B. The radial tuberosity is the site for tendinous attachment of the biceps brachii muscle, which supinates and flexes the forearm. When the tuberosity is destroyed, the biceps brachii is paralyzed.

65. Destruction of this area would most likely cause weakness of supination and flexion of the forearm.

The answer is E. The medial epicondyle is the site of origin for the common flexor tendon and pronator teres. The common flexors include the flexor carpi radialis and ulnaris and palmaris longus muscles, which can flex the elbow and wrist joints. Thus, destruction of this area causes weakness of pronation because the pronator teres is paralyzed but the pronator quadratus is normal. Similarly, destruction of this area causes paralysis of the flexors of the wrist. However, it can be weakly flexed by the flexor pollicis longus, flexor digitorum superficialis, and profundus muscles.

66. Destruction of this area would most likely cause weakness of pronation of the forearm and flexion of the wrist joints.

The answer is D. The olecranon is the site for insertion of the triceps brachii, which is innervated by the radial nerve. When the olecranon is destroyed, the triceps brachii is par- alyzed.

67. A lesion of the radial nerve would most likely cause paralysis of muscles that are attached to this area.

The answer is C. The hook of hamate and the pisiform provide insertion for the flexor carpi ulnaris.

68. Destruction of the structure indicated by the letter E most likely causes weakness of which of the following muscles? (A) Flexor carpi radialis (B) Palmaris longus (C) Flexor carpi ulnaris (D) Brachioradialis (E) Flexor digitorum superficialis

The answer is B. The scaphoid forms the floor of the anatomic snuffbox and provides a site for origin of the abductor pollicis brevis.

69. If the floor of the anatomic snuffbox and origin of the abductor pollicis brevis are damaged, which of the following bones is most likely to be involved? (A) A (B) B (C) C (D) D (E) E


Conjuntos de estudio relacionados

1.1.6 describe the cardiac cycle (atrial systole, ventricular systole and diastole); The structure and operation of the mammalian heart related to its function, including the major blood vessels.

View Set

OCS - Lumbar spine, OA, Pelvis, SI Coccyx and Abdomen

View Set

The Federal Reserve and Monetary Policy

View Set

COSC 102: Exam 1 Review - Sorting

View Set

Unit 6 (Ryan) African Americans in the British New World

View Set

Chapter 24 recall questions (BIRD)

View Set

COMPLETE RESEARCH METHODS STUDY PACK: ALL QUESTIONS FROM THE TEXT

View Set

Organizational Behavior Final Exam

View Set